Download PGI PG PGI 2017 November Solved Question Paper

Download PGIMER (Post Graduate Institute of Medical Education & Research, Chandigarh) PGI 2017 November Solved Question Paper


1. Muscle's of anterior compartment of leg is/
are:

a) Peroneus tertius
b) Peroneus brevis
c) Peroneuslongus
d) Flexordigitorumlongus
e) Flexor hallucis longus
Correct Answer - A
Ans. (a) Peroneus tertius
The 4 muscles in the anterior compartment of the leg are- the tibialis
anterior, extensor digitorum longus, extensor hallucis longus, and
fibularis(Peroneus) tertius




2. Not TRUE statement regarding parotid
gland is/are?

a) Deep lobe contains deep lymphatics
b) Divided into superficial and deep lobes by facial nerve
c) Parotid duct opens opposite to the second upper molar
d) Ectodermal in origin
e) Auriculotemporal nerve is the main sensory nerve
Correct Answer - A
Ans. is 'a' i.e., Deep lobe contains deep lymphatics
Development:
Parotid gland is the first salivary gland to appear, in early 6' week.
It is ectodermal in origin and develops from the buccal epithelium
just lateral to the angle of mouth
Structures emerging from parotid
The following structures emerge from the parotid gland:
Anterior border:
Parotid duct
3 Terminal branches of facial nerve:
The zygomatic and buccal branches: toward the temporal region,
eyelids and cheek, respectively.
Mandibular branch : Run along the body of the mandibletowards the
mouth
Apex:
5th terminal branch of facial nerve: Cervical branch continues into
the neck (to platysma).
Anterior & posterior divisions of retromandibular vein

Posterior border:
Posterior auricular nerve
Posterior auricular artery
Posterior auricular vein
Along base:
superficial temporal artery
temporal branch of facial nerve
Auriculotemporal nerve
STRUCTURES WITHIN GLAND:
Arteries:
External carotid artery enters through posteromedial surface
Maxillary artery
Superficial temporal vessel
Posterior auricular artery
Veins:
The retromandibular veins
Facial Nerve
Parotid Duct (Stenson's duct)
The duct turns opens into the vestibule of the mouth (gingivo- buccal
vestibule) opposite the crown of the upper 2nd molar tooth
Nerve supply:
PARASYMPATHETIC:auriculo temporal nerve
SYMPTHETIC SUPPLY- plexus around the external carotid artery.
SENSORY NERVES: auriculotemporal nerve, except for parotid
fascia & overlying skin which are innervated by Great auricular
nerve (C2, C3).


3. True about trochlear nerve:

a) Arise from ventral aspect of brainstem
b) Enters orbit through annulus of Zinn
c) Lesion causes diplopia
d) Nucleus of the trochlear nerve is located in the caudal
mesencephalon beneath the cerebral aqueduct
e) Damage causes ipsilateral palsy of superior oblique muscle
Correct Answer - C:D
Ans. (c) Lesion causes diplopia, (d) Nucleus of the
trochlear nerve is located in the caudal mesencephalon
The trochlear nerve has certain unique features:

It is the only cranial nerve whose fibers originate totally from the
contralateral nucleus.
It is the only cranial nerve to emerge from the dorsal surface of the
brain stem.
It is the most slender of all the cranial nerves.
It has the longest intradural course among the three extraocular
motor nerves.
It supplies only one muscle i.e. superior oblique (Abducent cranial
nerve also supplies only one muscle i.e. Lateral rectus).


4. True regarding thyroid gland is ?

a) Deep investing layer form Berry ligament
b) Condensed fibrous part of gland form true capsule
c) Superior thyroid artery lies posterolateral to superior
laryngeal nerve
d) Recurrent laryngeal nerve has variable course on both sides
e) Supplied by Thyrocervical Trunk
Correct Answer - B:E
Ans. Ans. is 'b' i.e., Condensed fibrous part of gland form
true capsule; 'e' i.e., Supplied by Thyrocervical Trunk
Thyroid gland has two capsule :-

. True capsule - peripheral condensation of the connective tissue
of the gland.
. False capsule - derived from the pretracheal layer of the deep
cervical fascia. It also forms the suspensory ligament of Berry
which connects the lobe to the cricoid cartilage.
. Ligament of berry and false capsule are derived from
pretracheal layer of deep cervical fascia (not investing layer).
Superior thyroid artery is related to external laryngeal nerve
(external branch of superior laryngeal nerve), but not directly
related to superior laryngeal nerve itself
Recurrent laryngeal nerve has variable relation to inferior thyroid
artery only on right side, on left side it has consistent relation
Blood supply of thyroid gland is through :
Superior thyroid artery - Branch of external carotid artery
Inferior thyroid artery - Branch of Thyrocervical trunk
Thyroidea ima artery - From the brachiocephalic trunk or arch of


aorta


5. Correct statement[s] regarding the
anatomy of vertebrae are ?

a) C7 has no foramen transversarium
b) C6 has prominent lateral mass
c) T3 has smallest spinous process
d) T12 has large costal facet
e) T7 has vertically oriented articular process
Correct Answer - E
Ans. e) T7 has vertically oriented articular process
C7 has foramen transversarium, but it does not transmits vertebral
artery (unlike foramina transversaria of C1 to C6)
Lateral mass is seen in C1 (not in C6)
Thoracic vertebrae (including T3) have long spinous process.
Small spinous processes are seen in cervical vertebrae (except for
C7).
T11 & T12 do not have costal facets
T2-79 are typical thoracic vertebrae and 71, TI0, 711, 712 are
atpical thoracic vertebrae
In typical thoracic vertebrae Articular processes are vertically placed
and interlocked; So dislocation can only occur if they are fractured.


6. Upper eyelid has following layer[s]-

a) Orbicularis oculi
b) Adipose tissue
c) Loose connective tissue
d) Levator palpebrae superioris
e) Muller muscles
Correct Answer - A:C:D:E
Ans.
is'a'i.e.,
Orbicularis
oculi;'c'i.e.,
Loose
connective
tissue;'d'i.e.,
Levator
palpebrae
superiors;'e'i.e., Muller muscles
Layers of eyelid :
Each lid is made up of (from without inwards)
Skin
Subcutaneous areolar tissue: Layer of loose areolar connective
tissue, without any fat.
Layer of striated muscles: Orbicularis oculi muscle ; In upper eyelid
levator palpebrae superioris
Submuscular areolar tissue :Nerves and vessels of the lids lie in this
layer
Fibrous layer: It is the framework of the lids and consists of :
. Tarsal plates: Tarsi form the skeleton of eyelids. Septum orbitale
and Muller's muscle are attached to superior border of Upper
tarsus. Orbital septum, capsulopalpebaral fascia and inferior
palpebral muscle are attached to inferior border of lower tarsus.
Tarial glands (meibomian glands) are embedded in the
posterior surface of tarsi.
. Septum orbitale (palpebral fascia): It is a thin, floating membrane


which takes part in all movements of eyelids' .
Medial and lateral palpebral ligaments.
Layers of non-striated muscle fibres: This layer consists of smooth
muscle fibers of muller muscles.
Conjunctiva


7. Branch [es] of ophthalmic artery is /are-

a) Central artery of retina
b) Supratrochlear artery
c) Anterior ethmoidal artery
d) Posterior ethmoidal artery
e) Posterior articular artery
Correct Answer - A:B:C:D
Ans.. is.a, i.e., Central artery of retina,'b'i.e., Supratrochlear
artery ,'c' i.e., Anterior ethmoidal artery 'd'i.e., Posterior
ethmoidal
Ophthalmic artery
It gives following branches :-
Central artery of retina ( end artery)
Lacrimal artery :- It gives following branches
? Lateral Palpebral branch. Zygomaticotemporal
Zygomaticofacial
Recurrent meningeal
Meningeal
Ciliary
Anterior ethmoidal-Supplies anterior ethmoidal sinus
Posterior ethmoidal
Medial Palpebral
Supratrochlear
Supraorbital
Dorsal nasal


8. Blood supply of optic tract comes through

a) Middle cerebral artery
b) Anterior cerebral artery
c) Posterior cerebral artery
d) Anterior choroidal artery
e) Internal carotid artery
Correct Answer - A:D:E
Ans .a, i.e., Middle cerebral artery 'd'i.e., Anterior
choroidal artery; 'e' i.e, Internal carotid artery Optic tract
is supplied by :

Anterior choroidal artery (branch of internal carotid artery)
Posterior communicating artery (branch of ICA);
Middle cerebral artery (Branch of ICA)


9. True about development of cochlea

a) Fully developed at 22 weeks
b) Derived from surface ectoderm
c) Develops in bony labyrinth
d) Develops from otic vesicle
e) Develops from otic capsule
Correct Answer - A:C:E
Ans. a. Fully developed at 22 weeks; 'c'i.e., Develops in
bony labyrinth; 'e' 1.e., Develops from otic capsule

Development of internal ear starts by the age of 3 weeks of
intrauterine life development of membranous internal ear is
completed by 16th week and reaches its adult size & shape by 20-
22 weeks when the cochlea is developed sufficiently.
Development of phylogenetically older part of labyrinth, i.e pars
superior (semi-circular canal and utricle) takes place earlier than
pars inferior (saccule and cochlea).
DEVELOPMENT DERIVATIVES OF EAR:
Surface ectoderm:
Membranous labyrinth
Epithelial lining of external auditory meatus
Outer surface of tympanic membrane
Mesoderm:
Bony labyrinth
Ear ossicle
Skeletal muscle
Centre of tympanic membrane
Endoderm:

Eustachian tube lining
Middle ear cavity
Epithelial covering of ossicle
Inner layer of tympanic membrane
Tympanic membrane is derived from all three layers of germ line.


10. Urogenital diaphragm is made up of ?
a) Deep transverse perinei
b) Perineal membrane
c) Sphincter Urethrae
d) Fascia of urogenital diaphragm
e) All the above
Correct Answer - E
Ans. e. All the above
Urogenital diaphragm :
Consists of two muscles :
Sphincter urethrae
Deep transverse perinei, also called transversus pernei profundus
Two fasciae:
Inferior fascia of urogenital diaphragm, also called perineal
membrane
Superior fascia of urogenital diaphragm


11. True about epiglottis ?
a) Contains serous gland
b) Contains mucous secreting glands
c) It is oval shaped
d) Made up of elastic cartilage
e) Has bilateral lymphatic supply
Correct Answer - B:D:E
Ans. b) Contains mucous secreting glands; d) Made up
of elastic cartilage ; e) Has bilateral lymphatic supply
Epiglottis:

The epiglottis of elastic cartilage tissue covered with a mucous
membrane, attached to the entrance of the larynx.
The epiglottis has two surfaces, lingual and a laryngeal surface,
related to the oral cavity and the larynx respectively
The entire lingual surface and the apical portion of the laryngeal
surface are covered by a stratified squamous non-keratinized
epithelium.
The rest of the laryngeal surface on, which is in relation to the
respiratory system' has respiratory epithelium: pseudostratified,
ciliated columnar cells and mucus secreting Goblet cells.
Epiglottis has bilateral lymphatic drainage topper deep cervical
lymph nodes


12. Which of the following statements are true
regarding diffusion of gas in lung -
a) Diffusion of gas is perfusion limited
b) CO, diffuses 20 times faster than 02
c) PO2 gradient is low between alveoli & blood vessel in case
of restrictive lung disease
d) Diffusion of gas is decreased in emphysema
e) Diffusion capacity of 0, is 25
Correct Answer - A:B:D:E
Ans. (A) Diffusion of gas is perfusion limited (B) CO, diffuses
20 times faster than 02 (D) Diffusion of gas is decreased in
emphysema (E) Diffusion capacity of 0, is 25

Ref: Textbook of respiratory physiology 3rd ed / p 913
Flow-limit (Perfusion - limited) Vs diffusion - limited transport:
Diffusion of O2, CO2, N2O across the respiratory membrane are all
flow (perfusion) limited.
CO transfer is diffusion-limited - Since carbon monoxide (CO) is
taken up by hemoglobin and it binds so avidly with Hb.
Diffusion Capacity Of O2 - 2O-25 ml/min/mmHg.
Diffusion Capacity Of CO2 - 400 ml/min/mmHg
CO2 diffuses 15-20 times faster than O2
Diffusion of gas in emphysema:
Emphysema Causes thickening of respiratory membrane
Causing reduced diffusion to gases.


13. Which of the following about renin
angiotensin system are true -
a) Renin is produced by modified smooth muscles of
glomerular capillary
b) Macula densa cells are part of this pathway
c) Angiotensinogen is converted to angiotensin by this pathway
d) Angiotensin converting enzyme is present in lung
capillary endothelial cell
e) Angiotensin -II is decapeptide
Correct Answer - A:B:C:D
Ans. (A) Renin is produced by modified smooth muscles of
glomerular capillary (B) Macula densa cells are part of this
pathway (C) Angiotensinogen is converted to angiotensin by
this pathway (D) Angiotensin converting enzyme is present
in lung capillary endothelial cell

Ref: Principles of medical physiology p. 417 Ganong 25th/e
p. 672-700

Juxtaglomerular apparatus:
The juxtaglomerular aPParatus is located at the angle of the afferent
and efferent arterioles, where it comes in contact with the distal
tubules.
It comprises the macula densa, juxtaglomerular (JG cells), and the
lacis cells.
Juxtaglomerular (granular) cells are modified smooth muscle cells in
the scala media of the terminal part of the afferent arterioles.
They contain large granules and secrete renin.

Lacis cells (extraglomerular mesangial cells) that are located in the
angular space between the junction of afferent and efferent
arterioles & also contain some renin.
The part of the distal tubule which comes in contact with the
afferent arteriole is made of a specialized epithelium called the
macula densa.
RENIN-ANGIOTENSIN SYSTEM:
Renin is a protease enzyme which is secreted by juxtaglomerular
(jG) cells of afferent arterioles.
Most powerful stimulus for renin release is reduced renal perfusion
pressure
Lowered pressure stimulate renin release.
Increased NaCl in distal tubules is sensed by macula densa and the
signal is transmitted to JG cells.
This results in decreased Renin release.
Opposite occurs when decreased NaCl is delivered in distal tubule,
i.e., increased renin release.
Adenosine is probably the mediator of signal.
JG cells are innervated by sympathetic fibers. They release renin in
response to sympathetic discharge, and by circulating
catecholamines.
The renin released from the jG cells enters the circulation and acts
on an alpha 2-globulin.
Angiotensinogen (secreted by the liver to convert it into a
decapeptide, angiotensin I by splitting Leucine-Valine bond
of angiotensinogen.
The enzyme angiotensin converting enzyme (ACE) then acts on
angiotensin I and convert it into an octapeptide, angiotensin II,
by splitting phenylalanine-histidine bond of angiotensin II.
ACE is found on the surface of capillary endothelium of lung,
therefore angiotensin II is formed on lung capillary endothelium.
Angiotensin II is degraded into angiotensin III by splitting
asparagine-arginine bond by aminopeptidase.


14. True about renal tubular system are ?
a) Ascending loop of Henle actively pumps Chloride out of tubule
b) Ascending loop of Henle actively pumps Carbonate out
of tubule
c) Descending loop of Henle is permeable to water
d) Ascending loop of Henle is impermeable to water
e) Descending loop of Henle receives hypotonic solution
Correct Answer - A:C:D
Ans. (A) Ascending loop of Henle actively pumps Chloride out
of tubule (C) Descending loop of Henle is permeable to water

(D) Ascending loop of Henle is impermeable to
water Ref: Ganong 25th/e p.680-682

Thin descending segment (limb):
Highly permeable to water.
Water absorption is obligatory and occurs through aquaporin-1 water
channel.
Relatively impermeable to solutes (sodium, chloride and urea).
Therefore, only water is reabsorbed from the thin descending Henle
hypertonic fluid.
No active secretion or reabsorption.
Thin ascending segment (Limb):
Less permeable to water but is very much permeable to Nacl.
Thick ascending segment (limb):
Totally impermeable to water.
Hypotonic to plasma.
Hence, Diluting segment.
30 % of filtered Na- is reabsorbed in ascending limb (60%,


reabsorbed in proximal tubule).


15. True statement about nerve muscle
physiology is ?
a) Action potential traverses along T tubules
b) Contraction is mainly because of extracellular calcium
c) Rynadine receptors sense the action potiential
d) actin pulls the myosin
e) ATP binds to myosin
Correct Answer - A:C:E
Ans. (A) Action potential traverses along T tubules
(C) Rynadine receptors sense the action potiential (E)
ATP binds to myosin


Excitation contraction coupling:
Skeletal muscle fiber is innervated by A-alpha neuron.
Carries the impulse (action potential) to neuromuscular junction
where release of acetylcholine from presynaptic vesicle occurs.
Events:
Each sarcomere has T-tubules.
T-tubule depolarizes, conformational changes occur in
dihydropyridine Receptors (DHPR) of T-tubules.
Leads to an interaction between DHPR and Ryanodine receptors
(RyR) in the terminal cisterns of sarcoplasmic reticulum.
DHPR-RyR interaction leads to release of Ca+ ions from the
terminal cisterns into the cytoplasm (sarcoplasm).
Diffusion of Ca+ into sarcoplasm causes muscle Contraction.
Molecular events:
Cross-bridge cycle:

Cross-bridging of myosin with actin produces bending (flexion) of
myosin head produces "Power stroke" sliding of actin on myosin
and muscle contraction.


16. True about copper metabolism is are ?
a) Intestinal absorption occurs in duodenum
b) 95% bond to albumin in the serum
c) Ceruloplosmin contains 6 atoms of copper
d) Failure to Synthesize ceruloplasmin causes wilson's disease
e) Copper excretion mainly occurs in urine
Correct Answer - A:C:D
Ans. (A) Intestinal absorption occurs in duodenum
(C) Ceruloplosmin contains 6 atoms of copper (D) Failure
to Synthesize ceruloplasmin causes wilson's disease


Ref: Dinesh puri 3ed/ p. 412
Copper primarily functions as a component of metalloenzymes or
proteins that participate in redox reactions.
Adult human body contains 50-100 mg of copper of which largest
amount are present in muscles (30-50mg), bones (10- 20mg),
and liver (10-15 mg).
Absorption:
Intestinal absorption of copper occurs mainly from duodenum.
Significance of Ceruloplasmin:
From portal circulation, copper is transported to liver, bind to
albumin,within hepatocytes copper is incorporated into
apoceruloplasmin to form ceruloplasmin.
Ceruloplasmin contains 6-8 atoms of copper.
Failure to synthesize ceruloplasmin is implicated in pathogenesis of
wilson disease.
Transport and excretion
Plasma concentration of copper is 100-200 mg/dl, about 95% of


which is bound to ceruloplasmin,
Excretion of copper mainly occurs in bile.


17. Which of the following enzymes of urea
cycle is/are not present in mitochondria?
a) Carbamoyl phosphate synthetase-1 [CPS-1]
b) Arginase
c) Arginosuccinase
d) Arginosuccinate synthase
e) Ornithine transcarbamylase
Correct Answer - B:C:D
Ans. is 'b' i.e., Arginase, 'c' i.e., Arginosuccinase & 'd' i.e.
Arginosuccinate synthase [Ref Harper's 30th/e p. 291 ]
Ammonia is ultimately disposed of by formation of urea by "Kreb's
Henseleit urea cycle" in the liver.
Urea cycle takes place both in mitochondria and cytosol.
First two reactions of urea cycle occur in the mitochondria, and
remaining reactions occurs in cytosol
Argininosuccinate synthase catalyzes the formation of
argininosuccinate from citrulline and aspartate. This reaction
requires IATP, but 2 high energy phosphate bonds are consumed
as ATP is converted to AMP + PPi. The amino group of aspartate
provides one of the two nitrogen atoms that appear in urea (The
other one is provided by ammonia NH4).
Argininosuccinate lyase (argininosuccinase) catalyses the cleavage
of argininosuccinate into arginine and fumarate. Fumarate enters
in TCA cycle.
Arginase catalyses the formation of urea from arginine by hydrolytic
cleavage of arginine to yield urea and ornithine. Ornithine is thus


regenerated and can enter mitochondria to initiate another round
of the urea cycle.


18. Regulatory enzymes of glycolysis are -
a) Phosphofructokinase
b) Pyruvate kinase
c) BPG kinase
d) Hexokinase
e) Glucose -6- phosphatase
Correct Answer - A:B:D
Ans. 'a' i.e., Phosphofructokinase, 'b' i.e., Pyruvate kinase & 'd'
i.e., Hexokinase [Ref Harper's 30th le p. 170-177]
Glycolysis is regulated at 3 steps which are irreversible. These
reactions are catalyzed by following key enzymes : (1)
Hexokinase and glucokinase, (2) Phosphofructokinase I, and (3)
Pyruvate kinase.
Hexokinase is found in most of the tissue except liver and comes
into play when blood glucose is low. It is not affected by feeding
or insulin or starvation. Hexokinase is not specific for glucose
metabolism, it is also involved in metabolism of fructose and
galactose.
Phosphofructokinase I is the major regulatory enzyme of glycolysis.
It catalyzes the 3rd reaction of glycolysis, i.e., fructose-6-P -a
Fructose 1,6 bis-P. This reaction is irreversible and is the "rate -
limiting step" for glycolysis.
It is allosterically activated by : Fructose-6-phosphate, fructose 2,6-
bisphophate, AMP, ADP, K. and phosphate. It is allosterically
inhibited by : ATP, citrate, Ca+2, Mg+2, and low pH.
Phosphofructokinase is an inducible enzyme that increases its


synthesis in response to insulin and decreases in response
to glucagon.
Pyruvate kinase is enzyme that catalyzes conversion of PEP to
pyruvate. Pyruvate kinase is an inducible enzyme that increases in
concentration with high insulin level and decreases with glucagon.
It is activated by fructose-1,6 bisphosphate and inactivated by ATP
and alanine.


19. Which of the following is/are to-6 fatty
acid(s)
a) Linoleic acid
b) Arachidinic acid
c) Gamma linolenic acid
d) Alpha linolenic acid
e) Oleic acid
Correct Answer - A:B:C
Ans. is 'a' i.e., Linoleic acid, 'b' i.e., Arachidinic acid & 'c' i.e.,
Gamma linolenic acid [Ref: Harper's 30th le p. 2141
Linoleic acid (18 carbon: Chemical formula 'aCH, - (CH,), - CH =
CH - CH2 - CH = CH - (CH2)7- COOH
C-System C:18:2:6,9'12 or C:18:2:9,12, i.e. 18 carbon fatty acid
with 2 double bonds at 915 (between C-9 and C-10) and 12'
(between C-12 and C-13) positions when numbering is started
from carboxyl carbon.
co-system C:18:2:w-6,9, i.e. 18 carbon fatty acid with 2 double
bonds at 6th (between C-6 and C-7) and 9' (between C-9 and C-10)
positions when numbering is started from terminal methyl carbon.
So linoleic acid is co-6 fatty acid, as the first double bond is at 6th
position in co-system classification.
Arachidonic acid (20 carbon ) Chemical formula C20H32O2


20. Nonreducing sugars are all except -
a) Glucose
b) Maltose
c) Sucrose
d) Fructose
e) Galactose
Correct Answer - A:B:D:E
Ans. is 'a' i.e., Glucose; b' i.e., Maltose; 'd' i.e., Fructose &
`e' i.e., Galactose [Rep Dinesh Puri 3"'/e le p. 24]

Reducing sugars are sugars which have free aldehyde or ketone
group their in their structure. Because of the presence of free
aldehyde or ketone group, they can reduce certain heavy metallic
cations in an alkali medium and in the process they themselves
get oxidized to a mixture of sugar acids.
Glucose and galactose have free aldehyde group at carbon-1, and
fructose has free ketone group at carbon-2. Thus, reducing end
of glucose and galactose is carbon-1 and of fructose is carbon-2.
Sucrose (disaccharide of glucose and fructose) is formed due to
formation of a-glycosidic bond between carbon-1 of glucose and
carbon-2 of fructose (Glucose - al 2 - Fructose). Thus, reducing
end of both glucose (carbon-1) and fructose (carbon-2) are
involved in glycosidic bond formation and therefore lost their
reducing property. Hence, sucrose is a non-reducing disaccharide.


21. Apo integrated in HDL is -
a) Apo Al
b) Apo E
c) Apo D
d) Apo B48
e) Apo B100
Correct Answer - A:B:C
Ans. is 'a' i.e., Apo Al; 'b' i.e., Apo E; & 'c' i.e., ApoD [Ref
Harper's 30th/e p. 257, 255; Chatterjee 6th/e p. 382]
Apo-A1 is found in HDL,Chylomicrons, site of synthesi is Liver,
intestine,and function in Major structural protein of HDL,
major activator of LCAT.
Apo-E (arginine rich), found in -Chylomicrons, chylomicron,site of
synthesis- liver, function as Mediates uptake of chylomicron
remnants and IDL by LDL receptors in liver.
Apo-D--found in HDL-Spleen, brain, testes, adrenal


22. Poor wound healing in vitamin 'C'
deficiency is due to all except -
a) Inhibition of collagen synthesis
b) Defective collagen synthesis
c) Defective post-translational modification of collagen
d) Defective hydroxylation
e) Defect in antioxidant system
Correct Answer - A
Ans. is "a" Inhibition of collagen synthesis
Ascorbic acid (Vitamin C) is also called antiscorbutic factor. It is very
heat labile, especially in basic medium. Ascorbic acid itself is an
active form. Maximum amount of vitamin C is found in adrenal
cortex.
Ascorbic acid functions as a reducing agent and scavanger of free
radicals (antioxidant). Its major functions are
. vitamin C is essential for the conversion of procollagen to collagen,
which is rich in hydroxyproline and hydroxylysine. Through collagen
synthesis, it plays a role in the formation of matrix of bone,
cartilage, dentine and connective tissue.
. Vitamin C is required for post-translational modification by
hydroxylation of proline and lysine residues converting them
into hydroxyproline and hydroxylysine.


23. UDP-glucose is used for -
a) Glycogen synthesis
b) Galactose metabolism
c) Bilirubin metabolism
d) Ganglioside synthesis
e) Heparin synthesis
Correct Answer - A:B:C:E
Ans. is 'a' i.e., Glycogen synthesis; 'b' i.e., Galactose
metabolism; 'c' i.e., Bilirubin metabolism; & `e' i.e.,
Heparin synthesis [Ref Basic medical biochemistry p. 475]

UDP-glucose is derived from glucose-6-phosphate via glucose-l-
phosphate.
The major fate of UDP-glucose is the synthesis of glycogen.
Other uses of UDP-glucose are -
. In uronic acid (glucoronic acid) cycle to generate UDP glucuronate.
. Galactose metabolism
Glycosylation of proteins, lipids and proteoglycans.
UDP glucuronate (which is derived from UDP glucose) is used for :-
. Conjugation of bilirubin, benzoic acid, sterols, estrogen and drugs.
. Biosynthesis


24. Which of the following enzymes have
proof reading function in PCR
[Polymerase Chain Reaction]

a) Taq polymerase
b) PFU Polymerase
c) Thermos thermophilus
d) Thermal flavus (Replinase)
e) T-7 polymerase
Correct Answer - B:E
Ans. is 'b' i.e., PFU Polymerase; & 'e' i.e., T-7 polymerase
[Ref Textbook of PCR by Mike McPherson]

The use of high fidelity DNA polymerases in PCR is essential for
reducing the introduction of amplification errors in PCR products.
Several thermostable DNA polymerases with 3' 5' exonuclease -
dependent proofreading activity have been introduced for high.
Pfu DNA polymerase Derived from Pyrococcus fusarious.
Pwo DNA polymerase Isolated from Pyrococcus woesei.
KOD HiFi DNA polymerase Isolated from Thermococcus
Kodakaraensis.
T7 DNA polymerase.


25. Which DNA polymerase has/have
proofreading activity -
a) DNA polymerase I
b) DNA polymerase II
c) DNA polymerase a
d) DNA polymerase
e) DNA polymerase y
Correct Answer - A:B:D
Ans. is 'a' i.e., DNA polymerase I; 'b' i.e., DNA polymerase II;
& 'd' i.e., DNA polymerase

DNA polymerase I - possesses three different
catalytic activities :?
. 5' 3' exonuclease activity, Polymerase activity (5'3'
polymerase activity), 3' 5' exonuclease activity,
. DNA polymerase I :- Helps in gap filling and synthesis between
okazaki fragments of lagging strand, and replaces ribonucleotides
of RNA primer by deoxyribonucleotides. It has (i) 3' 5'
exonuclease activity, (ii) 5' 3' exonuclease activity and (iii)
polymerase (5' 3' polymerase) activity.
. DNA polymerase I :- Helps in gap filling and synthesis between
okazaki fragments of lagging strand, and replaces ribonucleotides of
RNA primer by deoxyribonucleotides. It has (i) 3' 5' exonuclease
activity, (ii) 5' 3' exonuclease activity and (iii) polymerase (5'
3' polymerase) activity.
. DNA polymerase II:- Helps in (i) proofreading (3' ? 5'
exonuclease activity), and (ii) DNA repair.

. DNA polymerase III :- It is the main enzyme that synthesizes
prokaryote DNA, i.e., synthesis of leading and lagging strand. It has
(i) 51 3' polymerase (or simply polymerase) activity for DNA
synthesis, and (ii) 3'--> 51 exonuclease activity for proofreading.


26. Components / genes involved in RISC
complex -
a) Pasha
b) Mi RNA
c) rRNA
d) Drosha
e) Dicer nuclease
Correct Answer - A:B:D:E
Ans. is 'a' i.e., Pasha; 'b' i.e., Mi RNA; 'd' i.e., Drosha; & 'e' i.e.,
Dicer nuclease [Ref Textbook of molecular biology p.7121]

RNA- induced silencing complex (RISC) is a multiprotein complex
that incorporates one strand of a double stranded small
interfering RNA (siRNA) or single stranded micro RNA (miRNA).
RISC uses the siRNA or miRNA as a template for recognizing
complementary mRNA.
Once RISC finds complementary strand of mRNA (with help of
miRNA or siRNA), it activates RNAase to cleave (degrade)
mRNA.
The RISC-loading complex (RLC) is the essential structure required
to load dsRNA fragments into RISC in order to target mRNA. The
RLC consists of dicer, the human immunodeficiency virus
transactivating response RNA-binding protein (TRBP) and
Argonuate 2.
A nuclear RNAase specific for dsRNA called Drosha acts with a
nuclear ds-RNA binding protien called DGCR in human (Pasha
in Dorsophila) and cleaves the hairpin region out of long
precursor RNA generating a pre-miRNA.




27. Which of the following is required for
unwinding of DNA -
a) Helicase
b) Primase
c) SSBP
d) Ligase
e) Topoisomerase
Correct Answer - A
Ans. is 'a' i.e., Helicase [Ref: Harper's 30thle p. 383 & 29th p. 367]
protein
function
DNA polymerases Deoxynucleotide polymerization
Helicases
Processive unwinding of DNA
Relieve torsional strain that results
Topoisomerases
from helicase induced unwinding
DNA primase
Initiates synthesis of RNA primers
Single-strand
Prevent premature reannealling of
binding proteins
dsDNA
Seals the single strand nick
between the nascent chain and
DNA ligase
Okazaki fragments on lagging
strand


28. 30S ribosome CONSIST of all except -
a) mRNA
b) ATP
c) GTP
d) Initiating factor and Elongating factor
e) None
Correct Answer - E
Ans. is None [Ref Lippincates 5th/e p. 436]
Ribosomes are large complexes of r-RNA and proteins with one
large and one small subunits.
The small subunit binds m-RNA, thus guiding interaction between
m-RNA codon and anticodon of t-RNA to read the genetic
information with exiquisite fidelity. Hence small subunit is
responsible for accuracy, whereas the large subunit catalyzes
the formation of peptide bond.
Ribosomes are composed of two unequal subunits.
Eukaryotic(80 S) ribosome is made up of 60 S and 40 S subunits.
Prokaryotic (70S) ribosome is made of 50S and 30S subunits.
60S subunit of eukaryotes contains 5 S rRNA, 5.8 S rRNA, 28S
rRNA and more than 50 polypeptides. 40 S subunit contains 18 S
rRNA and about 30 polypeptide chains.


29. 30S ribosome INTERACTS with all
except?
a) mRNA
b) ATP
c) GTP
d) Initiating factor
e) Elongating factor
Correct Answer - B:E
ANS- 'b' i.e., ATP; & 'e' i.e., Elongating factor [Ref Dinesh Purl
Yale p. 482, 483]
INIATIATION process begins by formation of 30S initiation complex
between 30S ribosomal subunit, mRNA and formyl mettRNA. GTP
serves as source of energy. Three initiation factors (IF1, IF2, IF3)
are also required for formation of this complex.
Then there is formation of 70S initiation complex by joining of 50S
and 30S subunits. In this complex, initiator tRNA occopies P-site
on the ribosome. `A'-site is still empty.
Elongation factors : EF-Tu, EF-Ts, EF-G.


30. Enzymes required for mRNA synthesis
is/are ?
a) RNA polymerase I
b) RNA polymerase II
c) Primase
d) Topoisomerase
e) Ligase
Correct Answer - B
Ans. is 'b' i.e., RNA polymerase II [Ref Herper's 30"'/e p.
390-400]

It catalyzes the synthesis of mRNA, small nuclear RNA (sn-RNA)
and miRNA.
These products are coded by class II gene, i.e. class II gene is
transcribed by mRNA.
Class II gene differ from class I and III in that one of its transcribed
products (mRNA) is translated into protein.


31. Complementary DNA differs from genomic
DNA in -
a) Has coded segments
b) Has introns
c) Has only exons
d) Uses reverse transcriptase
e) Larger
Correct Answer - A:C:D
Ans. is 'a' i.e., Has coded segments; 'c' i.e., Has only exons; &
'd' i.e., Uses reverse transcriptase. [Ref: Lehninger 5"Ye p.
940-9601

Collection of cloned (recombinant) DNA fragments is called DNA
library or shotgun collection. DNA libraries may be of two types ?
1) Genomic library :-
The entire genomic DNA (both exons and introns) of an organism is
cut into small pieces by restriction endonucleases.
Each and every fragment is then cloned with suitable vector. These
recombinant clones are then collected.
2) Complementary DNA (cDNA) library :-
In cDNA library only exons are represented. It is constructed so as
to include only those genes that are expressed.
cDNA library is more specialized and exclusive DNA library. The
mRNAs from an organism is extracted and complementary double
stranded DNAs (cDNAs) are produced from these mRNAs by reverse
transcriptase. The resulting DNA fragments are then inserted into a
suitable vector and cloned




32. Micro Satellite instability is seen in ?
a) Huntington's disease
b) Lynch syndrome
c) Spinocerebellar ataxia
d) HNPCC
e) Colorectal cancer
Correct Answer - A:B:C:D:E
Ans. is 'a' i.e., Huntington's disease; 'b' i.e., Lynch syndrome;
'c' i.e., Spinocerebellar ataxia; 'd' i.e., HNPCC; & 'e' i.e.,
Colorectal cancer [Ref Textbook of genetic counselling p. 712]

Microsatellite instability is genetic instability in short nucleotide
repeats (microsatellites) due to high mutation rate as a result
of defects in mismatch repair of DNA.
Sometimes replication errors escape the proofreading function
during DNA synthesis causing a mismatch of one of several
bases. These errors are repaired later.
An important point is that the repair system must be able to
discriminate between the parent (template) strand and the new
daughter strand because it is the base on the daughter strand
parent that is incorrect so needs to be excised.
Methyl group on parent strand attached to adenine (methylated
adenine) near the mismatch serves as a tag by which the
repair system identifies .
Several Neurological disease are characterized by microsatellite
sequence instability including Huntington's disease, myotonic
dystrophy, Fragile-X syndrome, Friedreich's ataxia and
spinocerebellar ataxia.




33. Features of irreversible cell injury are
a) Lysosomal injury
b) Pyknosis
c) Cell membrane injury
d) Mitochondrial amorphous deposits
e) Apoptotic bodies
Correct Answer - B:C:D:E
Answer- B,Pyknosis C,Cell membrane injury
D,Mitochondrial amorphous deposits E,Apoptotic bodies
Characteristic features are -

Large flocculent amorphous densities in mitodrondria due to
accumulation of calcium.
Intracytoplasmic myeline figures appear during reversible injury but
becomes more prominent in irreversible injury.
Nuclear changes : These are most specific and include pyknosis
(nuclear condensation), karyorrhexis (fragmentation of nucleus),
and karyolysis (nuclear dissolution).
Decreased basophilia (due to decreased ribonucleo protein).
Leakage of intracellular enzyme across damaged cell membrane
into periphral circulation
Apoptotic bodies are seen in apoptosis, which is a pattern of death
after irreversible injury.


34. Premalignant lesion of oral cavity
includes
a) Lichen planus
b) Erythroplakia
c) Bowen disease
d) Behchet disease
e) None
Correct Answer - B
Answer- B. Erythroplakia
Premalignant condition: - Leukoplakia, Erythroplakia, Speckled
erythroplakia, chronic hyperplastic candidiasis.


35. Prothrombin time is elevated in following
conditions
a) Defect in factor XI
b) Fibrinogen defect
c) DIC
d) Factor VII defect
e) Von Willebrand disease
Correct Answer - B:C:D
Answer- B,Fibrinogen defect C,DIC D,Factor VII defect
It tests the extrinsic and common coagulation pathways. So, a
prolonged PT can results from deficiency of factor V,
VII, X, prothrombin or fbrinogen.


36. Following findings are seen in rheumatic
heart disease
a) Mc Callums plaque
b) Thickening of mitral valve
c) Fibrous plaque on undersurface of aortic valve
d) Aschoff bodies in myocardium
e) None
Correct Answer - A:B:D
Answer- A,Mc Callums plaque B,Thickening of
mitral valve D,Aschoff bodies in myocardium 1)
Acute rheumatic carditis-

The characteristic histological finding of rheumatic carditis is Aschoff
bodies/or Aschoff nodules.
2) Chronic rheumatic carditis-
Irregular thickening of posterior wall of left atrium produces
MacCallum plaque/patch due to subendothelial collection of
Aschoff nodules.
Endocardium involvement leads to formation of small warty
projections (verrucae) along the line of closure of valvular
leaflet, mostly on mitral valve.


37. Increased reticulocyte count is seen in
a) Megaloblastic anemia on treatment with hematinics
b) Acute hemorrhage
c) Congenital dyserythropoietic anemia
d) Hereditary spherocytosis
e) Aplastic anemia
Correct Answer - A:B
Answer- A,Megaloblastic anemia on treatment
with hematinics B,Acute hemorrhage

Acute blood loss or hemorrhage
Postsplenectomy
Microangiopathicanemia
Autoimmune hemolytic anemia
Hemoglobinopathy
Post anemia treatment
vitamin B12 supplementation


38. Increased reticulocytes are seen in
a) Aplastic anemia
b) B12 deficiency on treatment with hematinics
c) Iron deficiency anemia
d) Hemolytic anemia
e) None
Correct Answer - B:D
Answer- B,B12 deficiency on treatment
with hematinics D,Hemolytic anemia

Acute blood loss or hemorrhage
Postsplenectomy
Microangiopathicanemia
Autoimmune hemolytic anemia
Hemoglobinopathy
Post anemia treatment
vitamin B12 supplementation


39. Parameters which are increased more
than normal in iron deficiency anemia
are

a) TIBC
b) Serum ferritin
c) Transferring saturation
d) Transferring receptors
e) None
Correct Answer - A:D
Answer- A,TIBC D,Transferring receptors
Serum level decrease
TIBC increase
Serum ferrritin decrease
Red cell protoporphyrin decrease
Serum transferring receptors protein increased. (STFR to log of
ferritin)


40. The following laboratory finding
differentiate anemia of chronic disease
from iron deficiency anemia

a) TIBC
b) Transferring saturation
c) Serum iron levels
d) Decreased utilization of endogenous ferritin
e) All of theabove
Correct Answer - A:B:D
Answer- A,TIBC B,Transferring saturation
D,Decreased utilization of endogenous ferritin

Haemoglobin- anaemia mild to moderate
Blood picture- microcytosis and hypochromic (but mostly normocytic
& normochromic)
Absolute values- MCHC is low
Reticulocyte count- low
Red cell survival- shorten lifespan of erythrocytes
Bone marrow- myeloid hyperplasia & increase in plasma cells.
Serum iron & TIBC- low
Serum ferritin- increased (most distinguishing feature of chronic
disorder)
Other plasma proteins- raised


41. True about dystrophic calcification -
a) Raised calcium level
b) Seen in dead/degenerative tissue
c) Seen in Sarcoidosis
d) Seen in atherosclerosis
e) Seen in rheumatic fever
Correct Answer - B:D:E
Answer- (B) Seen in dead/degenerative tissue (D) Seen
in atherosclerosis (E) Seen in rheumatic fever
Dystrophic calcification in dead tissues-

Necrosis of tuberculosis (most common which may be in lymph
nodes)
Chronic abscess in liquifactive necrosis
Infarct
Thrombi
Dystrophic calcification in degenerated tissues
Atheromatous plaque
Psommama bodies
Heart valves damaged by rheumatic fever


42. Which of the following is/are due to non-
disjunction of autosomes
a) Klinefelters syndrome
b) Turners syndrome
c) Pataus syndrome
d) Edward syndrome
e) Cri du Chat syndrome
Correct Answer - C:D
Answer- C,Pataus syndrome D,Edward syndrome
Nondisjunction of autosomes: Down syndrome, edward syndrome,
patau syndrome


43. Which of the following acute phase
protein(s) decreases during acute
inflammation

a) Albumin
b) Transferrin
c) Ceruloplasmin
d) C-reactive protein
e) Haptoglobin
Correct Answer - A:B
Answer- A,Albumin B,Transferrin
Negative acute phase proteins
These proteins are decreased during inflammation. Important
examples are albumin, prealbumin, transferrin, transcortin,
transthyretin and retinal binding protein


44. Which of the following combinations are
true
a) Hyaline casts- normal
b) Waxy casts-chronic pyelonephritis
c) Broad casts - CRF
d) RBC casts-glomerulonephritis
e) Muddy casts-acute tubular necrosis
Correct Answer - A:B:C:D:E
Answer- A,Hyaline casts- normal B,Waxy casts-chronic
pyelonephritis C,Broad casts - CRF D,RBC casts-
glomerulonephritis E,Muddy casts-acute tubular
necrosis Hyaline casts

These are the most common casts and consists almost entirely of
Tamm-Horsfall protein
Renal tubular epithelial cell casts
Epithelial cells are present along with cast
Waxy casts
These are seen in chronic renal diseases
Broadcast
These are seen in advanced renal failure
RBC casts
There are seen in nepahritic syndrome due to glomenulonephritis
Muddy casts
It is pathognomonic of acute tubular necrosis (ATN)


45. True regarding cast examination in urine
a) Acid is added
b) Examined at the edge of cover slip
c) Sediment is used
d) Contrifuge is not used
e) Broad cast indicates stasis of urine flow
Correct Answer - A:C:E
Answer- A,Acid is added C,Sediment is used E,Broad
cast indicates stasis of urine flow

The cellular elements are best preserved in acid.
The urine sediment can be broken down into cellular elements.
Broad casts
Formation occurs in collecting tubules: serious kidney disorder,
extreme stasis of flow.


46. True about alcoholic steatosis
a) Microvesicular
b) Macrovesicular
c) Reversible
d) Mallory hyaline
e) Central hyaline sclerosis
Correct Answer - A:B:C
Answer- A,Microvesicular B,Macrovesicular
C,Reversible Hepatic steatosis (Fattyliver)

Initially there is microvesicular fatty change. Later macrovesicular
fatty changes are also seen. It is reversible stage.


47. True about Dubin-Johnson syndrome -
a) Increased conjugated bilirubin
b) Usually associated with increased AST and ALT
c) Mutation in uridine diphosphate-glucuronyltransferase
peptide Al
d) Decreased biliary excretion of conjugated bilirubin
e) Autosomal dominant inheritance
Correct Answer - A:D
Answer- (A) Increased conjugated bilirubin (D)
Decreased biliary excretion of conjugated bilirubin

During Johnson syndrome is an autosomal recessive hereditary
disorder presenting with conjugated hyperbilirubinemia due to
defect in hepatic excretory function across the canalicular
membrane of hepatocyte.
DJS is a type of congenital conjugated hyperbilirubinemia.
Conjugated bilirubin is increased because of defective biliary
excretion of bilirubin glucuronides due to mutation in canalicular
multi drug resistance protein 2.


48. Malignancies associated with HIV are -
a) Kaposis sarcoma
b) NHL
c) Anal Carcinoma
d) Cervical Carcinoma
e) Colon Carcinom
Correct Answer - A:B:C:D
Answer- (A) Kaposis sarcoma (B) NHL (C) Anal Carcinoma
(D) Cervical Carcinoma
Kaposi sarcoma is the most common tumor in AIDS.
Non-Hodgkin lymphomas (NHLs) in AIDS are -
. Primary CNS lymphoma (associated with EBV)
. Burkitt's lymphoma
Hodgkin's disease
Leukemia
Multiple myeloma
Cervical Ca
Anal Ca


49. Common metaphyseal tumors are -
a) Enchondroma
b) Osteosarcoma
c) Non ossifying fibroma
d) Osteoid osteoma
e) Osteoclastoma
Correct Answer - A:B
Answer- (A) Enchondroma (B) Osteosarcoma
Osteogenic sarcoma
Unicameral (simple) bone cyst
Aneurysmal bone ryst o Fibrous cortical defect
Chondrosarcoma
Osteochondroma
Enchondroma
Osteoblastoma


50. Which of the following combinations of
carcinoma with their genetic mutations
are true -

a) Carcinoma breast- BRCA1
b) Rhabomyosarcoma-C-KIT
c) Wilms tumor WT1
d) Retinoblastoma Rb
e) Neuroblastoma MYC
Correct Answer - A:C:D:E
Answer- (A) Carcinoma breast- BRCA1 (C) Wilms tumor WT1
(D) Retinoblastoma Rb (E) Neuroblastoma
MYC Carcinomas of female breast and ovary-
BRCA1 Wilms tumor- WT-l

Retinoblastomas- RB

Gastrointestinal stromal tumors, testicular seminoma, melanoma,
AML- C-KIT
Neuroblastoma- N MYC


51. Amydoid associated protein [AA protein]
is seen in -
a) Multiple myeloma
b) Dialysis associated amyloidosis
c) Systemic sclerosis
d) Sjogren's syndrome
e) Renal cell carcinoma
Correct Answer - E
Answer- E Renal cell carcinoma
. Primary
. Secondary (reactive)
There is deposition of AA amyloid protein-
Renal cell carcinoma (hypernephroma), Hodgkins lynphoma.


52. True about amyloid SSA -
a) Mutant transthyretin
b) Senile cardiac amyloidosis
c) Wild transthyretin
d) Familial polyneuropathy
e) Senile systemic amyloidosis
Correct Answer - B:C:E
Answer- (B) Senile cardiac amyloidosis (C) Wild transthyretin
(E) Senile systemic amyloidosis
SSA is characterized by depositio n of wild - type transthyretin (TTR)
- based amyloid in parenchymal organs in elderly individual.
SSA is common disease, affecting approximately 25%o of the
population greater than 80 years old.
SSA is characterized by amyloidosis clinically limited to heart;
therefore, initially it was referred to as senile cardiac amyloidosis.
This form of cardiac amyloidosis tends to run a benign clinical
course.


53. Feature(s) of Turner syndrome is/are -
a) Monosomy of autosomes
b) Webbing of neck
c) Mental retardation
d) Short fourth metacarpal
e) Streak gonades
Correct Answer - B:D:E
Answer- (B) Webbing of neck (D) Short fourth metacarpal
(E) Streak
gonades 45X0

Lymphadema of dorsum of hand & fat

Loose skin fold at nape of neck

Short stature

Short Neck (with webbing of neck)

Anomalies ear

Broad shield like chest with widely spaced small nipple

Renal anomalies (Horse-shoe, souble or cleft renal pelvis)Coart of
aorta


54. Definitive Risk factors for carcinoma
stomach is/are -
a) Smoking
b) Alcoholism
c) H Pylori infection
d) Chronic atrophic gastritc
e) Partial gastrectomy
Correct Answer - A:C:D:E
Answer- (A) Smoking (C) H Pylori infection (D) Chronic
atrophic gastritc (E) Partial gastrectomy

Environmental factors: H. Pylori infection, cigarette smoking, and
low socioeconomic status.
Host factors : Chronic gastritis, partial gastrectomy
lntestinal metaplasia is the most significant precursor lesion for
Gastric cancer.
Genetic factors


55. True about Digeorge Syndrome -
a) B-cell deficiency
b) Defect in 3rd pharyngeal pouch
c) Hypoparathyroidism
d) Candidiasis
e) Thymic aplasia
Correct Answer - B:C:D:E Answer- (B) Defect
in 3rd pharyngeal pouch

(C) Hypoparathyroidism (D) Candidiasis (E) Thymic aplasia
Digeorge syndome is an example of a T cell deficiency that results
from failure of development of the third and fourth
pharyngeal pouches.
Clinical features include-
Enhanced susceptibility- viral, fungal (mucocutaneous canididiasis)
and bacterial infections
Facial abnormalities : Hypertelorism, abnormal ears, short philtrum
and micrognathia
Hypocalcemic tetany due to failure of parathyroid development
(Hypoparathyroidism).


56. Prognosis of Head & neck cancer is based
on -
a) Site of the tumor
b) Stage of the tumor
c) Etiological agent
d) Age of patient
e) Gender of patient
Correct Answer - A:B:C
Answer- (A) Site of the tumor (B) Stage of the tumor
(C) Etiological agent
head and neck cancer is determined by tumor location and stage
and etiology.


57. Suspicion of malignancy in thyroid nodule
is indicated by all except -
a) Female gender
b) Dysphagia
c) Age 20-40 years
d) Increasing pain
e) Rapidly enlarging size
Correct Answer - A:B:C:E
Answer- (A) Female gender (B) Dysphagia (C) Age 20-40 years
(E) Rapidly enlarging size
The most common presenting sign of thyroid cancer is a thyroid
nodule.
Solitary or Multiple thyroid nodules
Neck Nodes
Hoarse voice of recent onset
Mediastinal adenopathy
Bone or lung metastasis
Gender: Female > Males.
Age:
More common at young adults.
MTC usually diagnosed after 60.
A history of a rapidly enlarging thyroid nodule usually indicates
hemorrhage, and this occur in both benign and malignant
disease.


58. True about competitive antagonists are ?
a) It decreases efficacy of agonist
b) It decreases potency of agonist
c) DRC is shifted to right side
d) Km is increased
e) Vmax is reduced
Correct Answer - B:C:D
Ans. (B) It decreases potency of agonist (C) DRC is shifted
to right side (D) Km is increased
Competitive inhibition:
km is increased.
V-max remains unchanged.
Competitive antagonist:
. Antagonist binds to the same receptors as agonist
. Antagonist resembles chemically with the agonist
. The same maximal response can be obtained
. Potency is reduced (Right shift of DRC)
. Km is increased but Vmax is unchanged
Non competitive antagonist:
. Binds to another site of receptor
. Does not resemble
. Maximal response is suppressed
. Efficacy is reduced (Flattening of DRC)
. Km is unchanged but Vmax. is reduced


59. Physiological antagonism is found in ?
a) Isoprenaline and salbutamol
b) Isoprenaline and adrenaline
c) Isoprenaline and propranolol
d) Adrenaline and histamine
e) Salbutamol and leukotrienes
Correct Answer - D:E
Ans. (D) Adrenaline and histamine (E) Salbutamol and
leukotrienes
[Ref KDT p. 56]
Physiological antagonists:
Produce opposite action by acting on different receptors.
Histamine - bronchoconstriction & adrenaline - bronchodilation.
Leukotrienes - bronchoconstriction & salbutamol - bronchodilation.


60. Side effect of clozapine are ?
a) Sedation
b) Seizures
c) Urinary incontinence
d) Decreased salivation
e) None
Correct Answer - A:B:C
Ans. (A) Sedation (B) Seizures (C) Urinary
incontinence [Ref : KDT p.429;Katzang /ep- 497-498]
Side effects of clozapine

Agranulocytosis
Urinary incontinence
Unstable BP & Tachycardia
Hypersalivation (sialorrhoea)
Worsening of diabetes
Weight gain
Seizures
Sedation


61. Which is true regarding naltrexone -
a) It is an opioid antagonist
b) It is an opioid agonist
c) Used in alcohol dependence
d) Used to treat opioid dependence
e) Used as a respiratory stimulant
Correct Answer - A:C:D
Ans. (A) It is an opioid antagonist (C) Used in alcohol
dependence (D) Used to treat opioid dependence
[Ref KD p.467; 433,353]
Naltrexone:
Pure opioid antagonist devoid of agonistic activity.
Recommended in alcohol dependence by US-FDA as adjuvant in
comprehensive treatment.
Used for 'opioid blockade' therapy of post-addicts.
Used to treat acute intoxication and maintenance therapy of opioid
withdrawal. However, it can precipitate withdrawal symptoms.


62. Which of the following is/are
cholinomimetic (Cholinergic) drug(s)?
a) Pilocarpine
b) Neostigmine
c) Bethanechol
d) Donepezil
e) Methacholine
Correct Answer - A:C:E
Ans. (A) Pilocarpine (C) Bethanechol (E)
Methacholine
[Ref KDT 7/e p. 104; Katzung 11*/e p. 98]
Cholinergic drugs may be divided into : (i) Directly acting, and (ii)
Indirectly acting (anticholinesterase).
Directly Acting:
With muscarinic action: Acetylcholine, methacholine, carbachol,
bethanechol, pilocarpine, muscarine, arecholine
With nicotinic action :- Acetylcholine, carbachol, arecholine (these
three drugs have both muscarinic and nicotinic effects).
Indirectly Acting (Anticholinesterase):
Two types
Carbamates: Physostigmine, Pyridostigmine, Ambenonium,
Galantamine, Neostigmine, Edrophonium, Donepezil, rivastigmine.
Organophosphates:
Echothiophate, Diazinon.


63. True about Carvedilol ?
a) al blocker
b) 31 blocker
c) 32 blocker
d) Antioxidant
e) Used in hypertension
Correct Answer - A:B:C:D:E
Ans. (A) al blocker (B) 31 blocker (C) 32 blocker (D) Antioxidant
(E) Used in hypertension
Carvedilol:

1 + 2 + 1 adrenoceptor blocker with - blocking property of 1:9.
Antioxidant and antimitotic ProPerty.
Produces peripheral vasodilation due to -1 blockade as well as
calcium channel blockade (direct effect).
Cardioprotective in CHF.
Causes orthostatic hypotension.


64. Interferon-alpha is used in the treatment
of ?
a) Hepatitis B
b) Hepatitis C
c) Multiple sclerosis
d) Chronic granulomatous disease
e) Multiple myeloma
Correct Answer - A:B:C
Ans. (A) Hepatitis B (B) Hepatitis C (C) Multiple sclerosis
[Ref. KDT P. 501, 802; Clinical pharmacologist P. 712]
Uses of interferon- are:
. CML
. Non-Hodgkin's lymphoma & cutaneous T-cell lymphoma
. Hairy cell leukemia
. Multiple myeloma
. AIDS related Kaposi sarcoma
. Chronic Hepatitis B & C
. HSV, HZV & CMV infection
. Rhinoviral cold
. Condyloma acuminata (HPV)
. Malignant melanoma
. Renal cell carcinoma


65. True about bedaquiline is ?
a) Not to be used in single line therapy
b) To be used in case of resistance to streptomycin
c) New unique antimicrobial introduced
d) Bactericidal drug
e) Used in TB resistant to rifampicin & isoniazide
Correct Answer - A:C:D:E
Ans. (A) Not to be used in single line therapy (C) New
unique antimicrobial introduced (D) Bactericidal drug (E)
Used in TB resistant to rifampicin & isoniazide

[Ref Katung 14/e p. 849; Lehn's Pharmacology ip. l081]
Bedaquiline (sirturo):
New antitubercular drug
Treatment of mulit-drug resistant tuberculosis
Works faster and better than all other anti-TB drugs.
Tuberculocidal.
By inhibiting ATP synthase.
No cross - resistance.
Pregnancy category B drug.
Uses:
Combination therapy for multidrug resistance TB (i.e. resistance to
rifampicin and lNH).
Not approved for treatment of latent TB, extrapulmonary TB or drug-
sensitive TB.
Adverse Effects:
Prolongation of QT interval.
Hepatotoxicity.




66. Extensive drug resistance TB is defined
as resistance to?
a) Amikacin
b) INH
c) Rifampicin
d) Pyrazinamide
e) Ciprofloxacin
Correct Answer - A:B:C:E
Ans. (A) Amikacin (B) INH (C) Rifampicin (E)
Ciprofloxacin
[Ref KDT 7th e p. 776]
Multidrug resistance (MDR) and extensive drug
resistance (XDR) TB:

MDR is defined as resistance to INH and rifampin with or without
resistance to other drugs.
XDR is defined as resistance to INH and rifampicin as well as to all
fluoroquinolones and one of injectable drugs (capreomycin,
kanamycin, amikacin).


67. Alternative to rifamicin based treatment of
leprosy in patient with hepatitis?
a) Clofazimine
b) Ofloxacin
c) Minocycline
d) Clarithromycin
e) Moxifloxacin
Correct Answer - A:B:C:D
Ans. (A) Clofazimine (B) Ofloxacin (C) Minocycline
(D) Clarithromycin
The World Health Organization (WHO) and the Indian Association of
leprologists have recommended an alternative combination therapy
of Clarithromycin, ofloxacin anil clofazimine to be prescribed in
such cases.
Minocycline can be used as an alternative to clarithromycin (i.e.,
minocycline, ofloxacin & clofazimine).


68. Anticancer drug(s) which does/do not
suppress bone marrow ?
a) 5-FU
b) Cisplatin
c) Chlorambucil
d) Vincristine
e) Vinblastine
Correct Answer - D
Ans. D. Vincristine
[Ref: Katzung p. 951-952]
Anticancer drugs with bone marrow sparing effect:
Vintistine
Bleomycin
Asparaginase


69. A patient has platelet count <1 lakh,
hemoglobin 8gm, which of the following
anticancer drug can be used in him ?

a) Cisplatin
b) Methotrexate
c) Vincristine
d) Vinblastin
e) Dleomycin
Correct Answer - C:D
Ans. (C) Vincristine (D) Vinblastin
Vincristine and bleomycin are bone marrow sparing drugs - can be
used in anemia and thrombocytopenia.


70. Which of the following drug(s) can cause
diarrhea?
a) Zinc
b) Ampicillin
c) Magnesium hydroxide
d) Aluminium hydroxide
e) None
Correct Answer - B:C
Ans. (B) Ampicillin (C) Magnesium hydroxide
Drugs causing diarrhea:
Broad spectrum antibiotics Digitalis Lactose
OCP
clindamycin
Magnesium antacids
Lincomycin
Purgative
Cocaine
Donepezil
Methyldopa
Reserpine
Colchicine Guanethidine
Misoprostol
Ticlopidine
AmPicillin


71. Selective norepinephrine (noradrenergic)
reuptake inhibitor(s) is/are?
a) Fluoxetine
b) Desipramine
c) Imipramine
d) Doxepin
e) Amoxapine
Correct Answer - B:E
Ans. (B) Desipramine (E) Amoxapine
Selective noradrenergic reuptake inhibitors (SNARIs)
Antidepressants which have high selectivity for noradrenergic
reuptake inhibitor over serotonin reuptake inhibition.
This group also includes the TCAs with predominant NA reuptake
inhibitory action.
Examples are - Desipramine, Nortriptyline, protriptyline,
Amoxapine, Reboxetine, Atomoxetine, maprotiline.


72. Management of NSAIDs toxicity include -
a) Sodium bicarbonate
b) Diazepam
c) Phenobarbitone
d) Hemodialysis
e) NH4C1
Correct Answer - A:B:C:D
Ans. (A) Sodium bicarbonate (B) Diazepam (C) Phenobarbitone
(D) Hemodialysis
[Ref, Principles of emergency medical care p. 301]
Ibuprofen is the most commonly used NSAIDs taken in overdose
followed by naproxen.
In acute overdose, activated charcoal can be used as
mechanical antidote.
Vomiting should be induced by Ipecac syrup, if the ingestion
occurred within minutes of arrival.
For dehydration oral or intravenous fluid should be given.
Metabolic acidosis will often respond to fluid resuscitation/ IV sodium
bicarbonate.
Convulsions are managed with IV benzodiazepines (diazepam,
lorazepam). Phenobarbitone is second choice if convulsions are
not controlled by BZDs.
Proton pump inhibitors (omeprazole etc) can be given for persistent
upper GI Symptoms.
Intubation may be required for airway protection due to coma or
prolonged uncontrolled convulsion activity.
Thiopentone is the DOC in these conditions.




73. True about first order kinetics of
elimination -
a) Constant amount of drug is eliminated
b) Rate of elimination is not related to plasma concentration
c) Clearance remains constant
d) Half life decreases with decreased concentration
e) None
Correct Answer - C
Ans. C. Clearance remains
constant
[Ref KDT p. 30]
First order kinetics:
Rate of elimination is directly proportional to Plasma Concentration.
Clearance remains constant -
Due to increasing plasma concentration, increases the rate of
elimination proportionately.
(CL = rate of elimination / Plasma Conc).
Half life remains constant -
Due to time required to reduce plasma concentration to half is same
(rate of elimination
change proportionately with plasma concentration).


74. Antibiotic(s) that require dose reduction in
renal failure?
a) Amikacin
b) Amphotericin B
c) Doxycycline
d) Rifampicin
e) Ceftriaxone
Correct Answer - A:B
Ans. (A) Amikacin (B) Amphotericin B
[Ref: Essentials of Pharmacology p 131]
Antimicrobials given after dose reduction:
Aminoglycosides
Ethambutol
Quinolones (except Grepa & trovafloxacin)
Cephalosporins (except cefoperazone & ceftriaxone)
Vancomycin
Amphotericin B
Antimicrobials contraindicated:
Nitrofurantoin
Nalidixic acid
Cephalothin & cephaloridine
Tetracyclines (except doxycycline)
Methanamine


75. True about sugamadex is/are -
a) It is a SRBA
b) Used to reverse rocuronium blockade
c) Used to reverse NM blockade
d) Use to reverse Sch blockade
e) Used in malignant hyperthermia
Correct Answer - A:B:C
Ans. (A) It is a SRBA (B) Used to reverse rocuronium blockade
(C) Used to reverse NM blockade
[Ref KDT e p. 355; Essentials of pharmacology p. 113]
Sugammadex:
Neuromuscular reversal drug,
1st in a new class of selective relaxant binding agent (SRBA) or
steroidal muscle relaxant encapsulators (SMRE).
Modified y-cyclodextrin with high affinity for aminosteroid non-
depolarizing muscle relaxants rocuronium and vecuronium.
Used to reverse neuromuscular block produced by rocuronium and
vecuronium (rocuronium > vecuronium).
Acts by forming a complex with muscle relaxant (rocuronium or
vecuronium).


76. 13 agonists are preferred over other
sympathomimetic drugs for -
a) Asthma
b) Uterine relaxation
c) Nasal decongesents
d) Orthostatic hypotension
e) Glaucoma
Correct Answer - A:B
Ans. (A) Asthma (B) Uterine relaxation
Clonidine and methyldopa are used in hypertension.
Other uses of clonidine are (i) preanaesthetic medication, (ii)
diarrhea in diabetic neuropathy, (iii) analgesic, (iv) withdrawal
syndrome of opoid, alcohol and nicotine, (v) prophylaxis of
migrain,
(vi) postmenopausal syndrome, (vii) suppression test for
pheochromocytoma, and (viii) for treatment of ADHD.


77. Which of the following joint is best
predictor of age of 16-17 year by X-ray:
a) Knee
b) Elbow
c) Hip
d) Wrist
e) Ankle
Correct Answer - D:E
Ans: (D) Wrist (E) Ankle [Ref Reddy 32nd/78, 77, 74; Parikh 7th/
61-63]
For estimation of age: Take X-ray
6-12 years Elbow joint, Wrist joint, 6 yr- center for lower end of
ulna (A); medial epicondyle of the humerus (A)
9 years: Olecranon (A)
9 to 11 years: Trochela of humerus (A)
10th to 11th year: Pisiform (A)
11th year: Lateral epicondyle of humerus (A)
13 to 16 years: X-ray of pelvis elbow joint &pelvis (13thyr -separate
centers in triradiate cartilage of acetabulum (A)
12 to 14 years: Lesser trochanter of femur (A)
14th year: Crest of ilium (A), fusion of medial epicondyle of
humerus, lateral epicondyle with trochlea
15th year: fusion of triradiate cartilage of acetabulum
16-17 year: wrist
Crest of ilium: 18-19.
Ischeal tuberostty,clavicle(inner end): 21-22(F), 23-24(M)




78. Gustafson's method is used for -
a) Age determination
b) Sex determination
c) Race determination
d) Teeth attrition
e) Teeth resorption
Correct Answer - A:D:E
Ans. is 'a' i.e., Age determination; 'd' i.e., Teeth attrition; `e' i.e.,
Teeth resorption [Ref Parikh 6th/e p. 2.8]
Gustafson's method : Useful only in persons older than 21 years
of age, depending on the physiological changes in each of the
dental tissues
. Attrition - due to wear and tear from mastication, upper surface of
teeth destoyed gradually, first involving the enamel --> dentine --
> pulp (depending on the functional use of teeth and hardness of
enamel). AO - no change. A - minimum change (enamel), A2 -
moderate change (dentine), A3 - maximum change (pulp).
. Paradentosis - recession of gums and periodontal tissue
surrounding the teeth, exposing the neck and adjacent part of
root --> teeth fall off (poor hygiene increases paradentosis).
. Secondary dentine formation - develop within the pulp cavity
and decrease size of the cavity, start from base - apex,
obliterate the cavity, increase with age, caries and paradentosis.
. Cementum apposition - near the end of root, increase cementum,
increase thickness, deposited throughout life, and form
incremental lines (devised by Boyde).
. Root resorption - because of cementum and dentine, absorption of


root start at apex and extend upward (may be pathological).
. Transparency of the root - seen after 30 years of age, canal in the
dentine at first widen, increase with age because of deposition of
minerals. They become invisible and dentine becomes
transparent (Most reliable of all the criteria).


79. Which of the following is NOT rape:
a) Sexual intercourse with wife, of age below 15 year, with
consent
b) Sexual intercourse with wife, of age 16 year, with consent
c) Sexual intercourse with a girl below 18 years of age, with
consent
d) Sexual intercourse with a girl of 18 year with consent
e) Sexual intercourse with wife who is living separately from him
under a decree of separation, or any custom or usage with her
consent
Correct Answer - B:D:E
Ans: b. Sexual intercourse with..., d. Sexual intercourse with
a girl..., e. Sexual intercourse with wife who is living ...... [Ref
Reddy 32nd/ 392-95; Parikh 7th/389-911

Rape: The Criminal Law (Amendment) Bill, 2013 (5.375,
I.P.C) Acc to Gazette Notification of GOI regarding Criminal Law
(amendendment) Act, 2013 released on 2 april, 2013, 1t is age
15 year .
Exception to S. 375, I.P.C: Sexual intercourse or sexual acts by
a man with his own wife, the wife not being under 15 years.
There is controversial reference regarding age of wife, either 16 or
15 year in Reddy (old & new ed.) & Parikh.
With her consent, when the man knows that he is not her husband
and that her consent is given because she believes that he is
another man to whom she is or believes herself to be lawfully
married.

With her consent when, at the time of giving such consent, by

reason of unsoundness of mind or intoxication or the administration
by him personally or through another of any stupefying or
unwholesome Substance, she is unable to understand the nature
and consequences of that to which she gives consent.

With or without her consent, when she is under eighteen years of
age.
When she is unable to communicate consent


80.


Unnatural Sexual offences are defined by
section ?

a) 375 IPC
b) 320 IPC
c) 377 IPC
d) 302 IPC
e) None
Correct Answer - C
Ans is 'c' i.e., 377 IPC [Ref Parikh Ole p. 3-68] IPC retated
to sexual offences]

228 IPC : Prohibits disclosure of identity of rape victim.
375 IPC : Defining rape.
376 IPC : Punishment for rape (7 years to life imprisonment ? fine).
376-A IPC : Punishment for marital rape (2 years imprisonment ?
fine).
377 IPC : Unnatural sexual offences (10 years to life imprisonment ?
fine).
354 IPC : Assault or criminal force to woman with intent to outrage
her modesty.
366-A IPC : Procuration of minor girl for illicit intercourse.
351 IPC : Defines assault.
352-358 IPC : Punishment for causing assault.
509 IPC : Word, gesture or act intended to insult the modesty of
a women.


81.


Sodomy is punishable under section IPC ?
f) 354
g) 375
h) 377
i) 378
j) None
Correct Answer - C
Ans is 'c' i.e., 377 [Ref Parikh 6"Ve p. 3.68]
Section 377 IPC defines all unnatural sexual offences (including
sodomy).
Section 377 IPC defines unnatural sexual sexual offences as
sexual intercourse against the order of nature with any man,
woman or animal and lays down punishment for same. These
offences are sodomy, buccal coitus, tribadism (lesbianism), and
bestiality.


82. Spalding sign is a feature of -
(E)
Drowning
(F)
Intrauterine death
(G)
Maceration
(H)
Rigor mortis
(I) Aseptic autolysis
Correct Answer - B:C:E
Ans is 'b' i.e., Intrauterine death; 'c' i.e., Maceration; `e'
i.e., Aseptic autolysis [Ref Parikh 6thie p. 5-75, 5-76]
Maceration:

Maceration is a process of aseptic autolysis. It occurs when a dead
fetus remains in the uterus for 3-4 days surrounded by liquor
amnii but with exclusion of air.
Skin slippage is the earliest sign (occurs within 12 hours).
There is gas in the great vessels and chambers of heart (Robert's
sign). Except for lung and uterus, which remain unchanged for a
long time, all other organs become soft, oedematous and loose
their morphology.
The one important radiological sign suggestive of maceration is
'Spaulding's sign' i.e. skull bones override each other. The smell
is somewhat rancid.
Rigor mortis:Rigor mortis may occur in dead fetus before birth or
at birth.
Putrefaction (decomposition) :If the membranes are ruptured
after death of fetus and air gains entry into liquor amnii, fetus
undergoes putrefaction instead of maceration. Body is greenish,
foul smelling and bloated




83. Extent of Burn is calculated by ?
16.
Rule of 9
17.
Wallace's formula
18.
Lund and Browder chart
19.
Henery's formula
20.
None
Correct Answer - A:B:C
Ans. is 'a' i.e., Rule of 9, `b i.e., Wallace's formula; 'c' i.e.,
Lund and Browder chart [Ref Parikh &le p. 4-152-4-156]

In adults surface area is calculated by :
f) 9% for the head and neck.
g)
9% for each upper limb.
h)
9% for the front of each lower limb.
i) 9% for the back of each lower limb.
j) 9% for the front of the chest.
k)
9% for the back of the chest.
l) 9% for the front of the abdomen.
m) 9% for the back of the abdomen.
n)
1% for the genitalia.
Area of palm (palmar surface), as a general rule, constitues about
1% of surface area both in adult and children.
In burnt area >15% in adults and >10% in a child, the loss of
circulatory blood volume must be replaced, otherwise it will
cause irreversible shock.
Burns on head, neck, trunk and genitals are said to be more
dangerous than on other parts of the body.
Infants, young children and elderly are particularly vulnerable to


initial shock and subsequent complications.


84. True about tapeworm is/are -
(D)
Taenia saginata is beef tapeworm
(E)
Taenia solium is more prevalent
(F)
Infection is acquired by ingestion of cysticercus in raw beef
(G)
Infectious form to animal is egg
(H)
None
Correct Answer - A:C:D
Ans. is 'a' i.e., Taenia saginata is beef tapeworm; 'c' i.e.,
Infection is acquired by ingestion of cysticercus in raw
beef;`d' i.e., Infectious form to animal is egg [Ref Medical
microbiology
E-book 215]

Two species of Taenia infect man :?
f) Taenia saginata : The beef tapeworm.
g)
Taenia solium : The pork tapeworm.
Mode of infection of tapeworms
h)
T. saginata - Under cooked beef containg cysticercus bovis.
i) T. solium Undercooked pork containg cysticercus cellulosae &
rarely by ingestion of egg (autoinfection).
Occasionally man gets infected in the same way as pig, by ingestion
of eggs (either by drinking contaminated water or by eating
uncooked vegetables infected with eggs). In this cycle man acts
both as definitive as well as intermediate host.


(D) Most common cause of non-gonococcal
urethritis -
18.
Mycoplasma hominis
19.
Mycoplasma genitalium
20.
Ureoplasma urealyticum
21.
Chlamydia trachomatis
22.
Haemophilus ducrey
Correct Answer - D
Ans. is 'd' i.e., Chlamydia trachomatis [Ref: Essentials
of microbiology 3rdle p. 786]
urethritis
Gonococcal
Nongonococcal
Neisseria gonorrhea
Chlamydia Trachomatis (most
common)
Ureoplasmaurealyticum
Mycoplasmagenitalium
Bacterioides
Haemophilus species
Candida albicans
T.vaginalis


86. Lesions caused by spirochetes are -
f) Syphilis
g) Yaws
h) Legionella pneumonia
i) Pinta
j) Lyme's disease
Correct Answer - A:B:D:E
Ans. is 'a' i.e., Syphilis; 'b' i.e., Yaws; 'd' i.e., Pinta; 'e'
i.e., Lyme's disease

Spirochetes are elongated, spirally coiled, flexible bacteria.
Characteristic feature of spirochetes is presence of endoflagella
which do not protrude outside.
Pathogenic spirochetes belong to three genera : Treponema,
Borrelia And Leptospira.
Spirochete species
disease
Treponema Pallidum
Syphilis
Endemicum Bejel
Pertunae
Yaws
Carateum Pinta
Borrelia
Burgdorferi Lyme disease
Recurrentis Relapsing fever
Vincenti
Vincent angina
Leptospira Interrogans Weil's disease


87. True about streptococcus agalactiae -
f) Catalase positive
g) Catalase negative
h) Beta hemolytic
i) Alpha hemolytic
j) Bacitracin resistant
Correct Answer - B:C:E
Ans. is 'b' i.e., Catalase negative; 'c' i.e., Beta hemolytic; 'e'
i.e., Bacitracin resistant

Streptococcus agalactiae is bacitracin resistant. Only streptococcus
sensitive to bacitracin is streptococcus pyogenes.
Sensitivity to bacitracin is employed as a convenient method for
differentiating str. pyogenes from other hemolytic streptococci
(Maxted's observation).
Streptococcus agalactiae (also known as group B streptococcus or
GBS) is a gram-positive coccus (round bacterium) with a tendency
to form chains (as reflected by the genus name Streptococcus). It
is a beta-hemolytic, catalase-negative, and facultative anaerobe.


88. True about cellulitis Vs erysipelas -
20.
Cellulitis is deep
21.
Erysipelas is deep
22.
Both caused by streptococcus pyogenes
23.
Cellulitis starts abruptly
24.
Erysipelas starts slowly
Correct Answer - A:C
Ans. is 'a' i.e., Cellulitis is deep; 'c' i.e., Both caused by
streptococcus pyogenes [Ref Text book of Dermatology p. 1531

Erysepelas and cellulitis are caused mostly by streptococcus
pyogenes
Cellulitis affects the deeper loose subcutaneous tissue. As in any
continum of disease, some overlap can occur. Despite their
common etiology, significant differences in presentation, signs, and
clinical course are noted.
Almost all cases erysipelas are caused by Str pyogenes. Whereas
cellulitis is caused most commonly by Str pyogenes, rarely it can
also be caused by staphylococcus, ldebsiella, H influenzae


89. True regarding erysipelas are -
f) Caused by S pyogenes
g) Superficial skin lesion
h) More seen on face than trunk
i) Has history of recent sore throat infection
j) None
Correct Answer - A:B:C:D
Ans. is 'a' i.e., Caused by S pyogenes; 'b' i.e., Superficial skin
lesion; 'c' i.e., More seen on face than trunk; i.d., Has history
of recent sore throat infection [Ref Text book of Dermatology
p.
1531

Erysipelas is a superficial skin infection affecting upper dermis and
extends into the superficial lymphatics.
It is caused mostly by streptococcus pyogenes (group A Beta
hemolytic streptococcus).
Symptoms and signs of erysipelas are usually abrupt in onset and
often accompanied by fevers, chills and shivering.
Erysipelas predominantly affects the skin of the lower limbs, but
when it involves the face it can have a characteristic butterfly
distribution on the cheeks and across the bridge of the nose.
The affected skin has a very sharp, raised border.
It is bright red, firm and swollen. It may be finely dimpled (like an
orange skin).
It may be blisthered, and in severe cases may become necrotic.
Bleeding into the skin may cause purpura.


f) Most common route of transmission of
tuberculosis to neonate -
f) Aerosol infection
g) Skin contact
h) Transplacental infection
i) Venous route
j) Haematogenous infection
Correct Answer - C:D:E
Ans. is 'c' i.e., Transplacental infection; 'd' i.e., Venous route; 'e'
i.e., Haematogenous infection [Ref Clinical neonatology 4thie
p.
153]

Transplacental spread:
It is the most common mode of transmission
Tubercle bacilli cross the placenta through the umbilical vein and a
primary focus develops in the fetal liver with involvement of
periportal lymph nodes and the bacilli infect secondarily the lung. It
is a hematogenous infection and is called congenital infection by
vertical transmission of TB.
Aspiration or ingestion
Neonate may acquire the disease in utero or during child birth
(intrapartum) by aspiration or ingestion of infected aminiotic fluid.
It causes primary infection of fetal lungs and gut.


23. Incorrect statement(s) regarding HPV is
are -
f) DNA virus
g) RNA virus
h) 16, 18 types cause carcinoma
i) Verrucus vulgaris is most common
j) Recently approved vaccine is trivalent
Correct Answer - B:E
Ans. is 'b' i.e., RNA virus; 'e' i.e., Recently approved vaccine
is trivalent

HPV is a non-enveloped DNA virus (ds DNA), belongs to
Papovaviridae.
HPV selectively infects the epithelium of skin and mucous
membrane and may immortalize the keratinocytes leading
either asymptomatic infection, or warts or neoplasia.
Products of E-genes (E6, E7) are related to immortalization or
malignant transformation of keratinocytes by interfering with p53
and Rb genes, respectively.
HPV infects only human skin and grows only in organ cultures of
human skin.
Warts are benign extra growth of skin and mucous membrane
resulting from infection with human papilloma virus (HPV). They
are common in children and young adults
There are two types of HPV vaccines :-
Quadrivalent :- containing HPV types 6,11,16,18
Bivalent :- containing HPV types 16,18

Efficacy of vaccine has varied according to immunologic and
virological characteristics of study populations at baseline and
according to the endpoints evaluated. Most of the time, rates
of vaccine efficacy exceed 90%.
Vaccine is recommended for girls and young women 9-26 years of
age.


92. Correct about adenovirus -
. Causes conjunctivitis
. Diarrhea is caused by serotype 40
. Single stranded DNA virus
. Causes intussusception
. Type 14 causes most severe disease
Correct Answer - A:B:D:E
Ans. is 'a' i.e., Causes conjunctivitis; 'b' i.e., Diarrhea is
caused by serotype 40; 'd' i.e., Causes intussusception & `e'
i.e., Type 14 causes most severe disease.

Non-enveloped, DNA (ds DNA) Virus
Has characteristic morphology consisting of an icosahedral shell
composed of 20 equilateral triangular faces and 12 vertices -3
space vehicle appearance.
Human adenovirus grows only in tissue culture of human
origin, such as human embryonic kidney, HeLa or HEP-2.
Cytopathic effects in tissue culture ---> cell rounding and
aggregation into grape like clusters.
Adenovirus has been used as a vector for gene therapy.
Adenovirus cause infections of the respiratory tract and eyes and
less often of the intestine and urinary tract.
Most frequently affect infants and children
Most common manifestation in children is an acute upper respiratory
tract infection with prominent rhinitis.
Most common manifestation in adult ARDS.
Adenovirus serotype 14 has been associated with severe and
potentially fatal outbreak of pneumonia.




.
All are true about Ebola virus infection
except?

f) Air dropler is most common mode of transmission
g) Haemorrhagic manifestation may occur
h) Thai forest type - most common species in epidemics
i) presents as sudden onset of fever and sore throat
j) Case fatality rate may be high as 70%
Correct Answer - A:C
Ans. is 'a' i.e., Air dropler is most common mode of
transmission & 'c' i.e., Thai forest type - most common

species in epidemics ]Ref Park 24m/e p. 374]
The virus is transmitted through direct contact with blood, organs,
body secretions or other body fluids of infected animals like
chimpanzees, gorillas, monkeys, fruit bats etc.
Human to human transmission is through blood or body fluids of an
infected symptomatic person or through exposure to objects (such
a needle) that have been contaminated with infected secretions.
It is not transmitted through air, water, or food.
The virus is transmitted through direct contact with blood, organs,
body secretions or other body fluids of infected animals like
chimpanzees, gorillas, monkeys, fruit bats etc.
Human to human transmission is through blood or body fluids of an
infected symptomatic person or through exposure to objects (such
as needle) that have been contaminated with infected secretions
It is not transmitted through air, water, or food
The illness is characterized by sudden onset of fever, intense


weakness, muscle pain, headache, sore throat, vomiting,
diarrhea, rash, impaired kidney and liver function and in some
both internal and external bleeding.
The virus family Filoviridae includes three genera: Cuevavirus,
Marburgvirus, and Ebolavirus.
Within the genus Ebolavirus, five species have been identified:
Zaire, Bundibugyo, Sudan, Reston and TalForest.
The first three, Bundibugyo ebolavirus, Zaire ebolavirus, and
Sudan ebolavirus have been associated with large outbreaks
in Africa.



94. CD marker on natural killer cells ?
. CD-16
. CD-56
. CD-3
. CD-4
. CD-8
Correct Answer - A:B:C
Ans. is 'a' i.e.,CD-16; `b' i.e., CD-56; 'c' i.e., CD-3 [Ref
Anantanarayan Sole p. 137; Harrison 19th/e p. 372]
Null cells are called so because they lack features of surface
markers of both B and T lymphocytes. The account for 5 to 10%
of peripheral blood lymphocytes.
They are also called "large granular lymphocytes (LGL)" as they
contain large azurophilic cytoplasmic granules. They express
CD16 and CD56 markers. They proliferate in response to IL-2.
LGLs arise in both bone marrow and thymic microenvironment. In
contrast to T-cells, they are not MHC restricted.
NK-cells are positive for CD16 and CD56.
NK cells are usually negative for CD3, but a subset is positive for
CD3 called NK/T-cells.
IL-2 acts as a growth factor for NK cells. NK cells activity is
augmented by interferon. NK cells secrete peroforin (resembles
comlement C9) which causes transmembrane pores through which
cytotoxic factors (e.g. TNF-(3) enters the cells and induce
apoptosis.


.
Deficiency of both B and T lymphocytes is
seen in ?

25.
Wiskott Aldrich syndrome
26.
Digeorge syndrome
27.
Ataxia telangiectasia
28.
Common variable immunodeficiency
29.
Chronic mucocutaneous candidiasis
Correct Answer - A:C
Ans. is 'a' i.e., Wiskott Aldrich syndrome; 'c' i.e.,
Ataxia telengectasia [Ref Atlas of immunology p. 537]
Combined immunodeficiencies (B and T cell defects)

Cellular immunodeficiency with abnormal immunoglobulin synthesis
(Nezelof syndrome)
Ataxia telangiectasia
Wiskott-Aldrich syndrome
Immunodeficiency with thymoma
Immunodeficiency with short-limbed dwarfism 0 Episodic
lymphopenia with lymphocytotoxin
Severe combined immunodeficiencies
'Swiss type' agammaglobulinemia
Reticular dysgenesis of de Vaal
Adenosine deaminase (ADA) deficiency


96. True about treatment of leprosy ?
f) Multibacillary leprosy is treated by 3 drugs
g) Paucibacillary leprosy is treated by 1 drug
h) Paucibacillary leprosy is treated for 1 year
i) Post-treatment surviellance for multibacillary leprosy is for
5 years
j) Clofazimine is first line drug
Correct Answer - D:E
Ans- d) Post-treatment surviellance for multibacillaryleprosy
is for 5 years; 'e' i.e., Clofazimine is first line drug [Ref

Harrison 19th le p. 11261 .
Paucibacillary : Bacteriological index is less than 2 with 5 or less
skin lesions. It includes Borderline tuberculoid (BT), Tuberculoid
(TT) and Indeterminate leprosy.
Multibacillary : Bacteriological index is 2 or more with 6 or more skin
lesions. It includes Borderline (BB). Borderline lepromatous (BL)
and Lepromatous (LL) leprosy.
Drugs used for leprosy are :-
First line : Rifampicin, dapsone and clofazimine.
Other : Minocycline, Quinolones (oxacilin), ethionamide,
clarithromycin, rifapentine and moxifloxacin.
Rifampicin is the most active (rapidly acting and bactericidal drug)
for leprosy


26. Painful penile ulcer with suppurative
lymphadenopathy is seen in ?
f) Syphilis
g) Hard chancre
h) Soft chancre
i) Haemophilus ducreyi
j) Donovanosis
Correct Answer - C:D
Ans. is 'c' i.e., Soft chancre; 'd' i.e., Haemophilus ducreyi
Chancroid (Soft chancre), Haemophilus Ducreyi*, 1-14 days, ?
Painful non indurated ulcer (soft chancre), Painful lymphadenopathy


98. Not true about candida albicans -
. Dimorphic fungus
. Yeast like fungus
. Reynolds-Braude phenomenon
. Forms true hyphae
. Forms pseudohyphae
Correct Answer - A:B:C:D:E
Ans. is 'None' i.e., All options are correct [Ref Ananthnarayan
Stile p. 607; Essentials of Microbiology p. 717
Candida is a yeast like fungus.
All candida species pathogenic for humans are also encountered as
commensals of humans, particularly in the mouth, stool and vagina.
They grow rapidly on simple media as oval budding cells at 25? to
37?C.
In tissue, both yeasts and pseudohyphae are present.
Candida albicans is differentiated by other candida :
It forms true hyphae (mycelia) or germ tubes when grown in serum.
It forms thick walled large spores called chlamydospores when
grown in corn meal agar.
It is dimorphic. Candida albicans can produce yeast, true hyphae
and pseudohyphae.
A rapid method of identifying C. albicans is based on its ability to
form germ tubes within two hours when incubated in human serum
at 37?C --> Reynolds - Braude phenomenon ( Also known as
germ tube test).
Candida yeast cells and pseudohyphae are the only fungi that are
usually gram - positive on smears.




99. True about aspergillus infection -
. Forms hyphae
. Branched at 90?
. Nonseptate
. Septate
. Causes subcutaneous infections
Correct Answer - A:D
Ans. is 'a' i.e., Forms hyphae; 'd' i.e., Septate [Ref
Ananthnarayan 8th/e p. 613; Harrison 19'Ve p. 1346]


.
Which of the following Nanoparticle(s)
inhibit both E coli and Staph aureus ?

. Zinc oxide
. Silver nanoparticle (SNP)
. Copper oxide
. Aluminum oxide
. None
Correct Answer - A:C
Ans. is 'a' i.e., Zinc oxide; 'c' i.e., Copper oxide [Ref
Clinical
Nanomedicine handbook]

A number of nanoscale metals, metal oxides, oral natural &
synthetic polymers possers antimicrobial properties.
ZnO Staphylococcus, Escherichia coli
ZnO ions Pseudomonas aeruginosa, S aureus Candida albleari
SNP E. coli, Vibrio cholera, Salmonella lyphi, P. aeruginosa
AI203 E. coli, B. subtilis, Pseudomonas fluorescens


27. True about six-sigma method in health
care?
f) Used to Improve patient care
g) Requires four steps
h) It reduces patient dissatisfaction
i) Collect and analyze the data
j) Can be used for administration as well
Correct Answer - A:C:D:E
Ans. is 'a' i.e., Used to Improve patient care; 'c' i.e., It
reduces patient dissatisfaction; 'd' i.e., Collect and analyze
the data; & `e' i.e., Can be used for administration as well
[Ref www.villanovau.com]

To help reduce waste and improve quality care, many hopitals and
healthcare practices have adopted Six sigma management tools to
help achieve goals. Employing Six Sigma principles in healthcare
settings can help eliminate defects and variation in processes, and
it can help make procedures more streamlined, less costly and help
improve patient care.
In healthcare environments a defect is defined as a factor that leads
to patient dissatisfaction. Examples of defects range from
the frustrating kind, such as a long wait to see a doctor, to the
serious kind, such as an incorrect diagnosis or treatment. Because
patient care is hands-on, the possibility for variance is larger than in
other process-driven industries. Variables may be small and difficult
to quantify, but Six Sigma's data-driven approach can result in
measurable improvement.

Six-Sigma is a quality improvement methodology that applies
statistics to measure and reduce variation in processes.
It is an organizational philosophy in establishing the belief of 'doing
things right first time and everytime'. It is 'a measure of quality that
strives for perfection'. The term Six Sigma means 'standard
deviation'.
In Health care sector it is used for improving quality of health care
services and reducing patient's dissatisfaction. It is used to
reduce the errors and to move towards


102. all are Steps used in Six-sigma except -
28.
Define
29.
Analyze
30.
Improve
31.
Feed back
32.
Control
Correct Answer - D
Ans- "D" Feed back
Six Sigma requires five steps for quality improvement?
f) Define -+ Define the problem, clarify and relate it to the custmer. Who
are the patient's, and what they want? What are the objectives?
f) Measure Measure your target metric and know your measure
is good. What will improvement look line? On what data will our
effect measured.
30. Analyze -+ Look for root cause and generate a prioritized
listing of them collect data and analyze using proven tools.
f) Improve Determine and confirm the optimal
solution Implement modification to improve the process.
31. Control Be sure the problem does not come back and
sustain it Monitor performance to maintain improvement.
Popular outcomes from Six - Sigma
Increased patient satisfaction and care, Fewer complaints,
Increased prescription, accuracy, Reduced waiting time and
variation, Safer and more efficient emergency services, Fewer
medical errors defects, Increased physician satisfaction.


f)
Causal association can be best
established in -

32.
RCT
33.
Cohort study
34.
Case control study
35.
Ecological study
36.
None
Correct Answer - A
Ans.'a'i.e.,RTC
As a single study unit, double blind RCT is the best.
However, overall meta-analysis is a better study since it combines
the data from multiple RCT and also from other types of study.
"Randomized controlled trials (RCT) provide the stongest, most
relevant evidence to inform practice. Some evidence hierarchies
place systematic review and meta-analysis above RCTs since
these often combine data from multiple RCTs, and possibly from
other study type as well" -- Epitemiology at glance
So, systematic review and meta-analysis of RCTs are best
epidemiological studies


f)
Which of the following is/are correct
regarding Cohort study Vs Case-control
study?

f) Case-control study is easy
g) Incidence is well calculated by case-control study
h) Cohort study can estimate both relative risk and attributable risk
i) Disease has not occured at start of cohort study
j) Cohort study is cheaper
Correct Answer - A:C:D
Ans. is 'a' i.e., Case-control study is easy; 'c' i.e., Cohort study
can estimade both relative risk and attributable risk & 'd' i.e.,
Disease has not occured at start of cohort study.
Case control study
Cohort study
1. Proceeds from "effect to cause"
2. Starts with the disease
1. Proceeds from "cause to effect"
f) Tests whether the suspected 2. Starts with people exposed to,risk
cause occurs more frequently in factor or suspected cause
those with the disease than 3. Tests whether disease occurs more
among those without the frequently in those exposed, than in
disease those not similar exposed.
35. Usually the first approach to the4. Reserved for testing of
precisely testing of a hypothesis, but also formulated hypothesis.
useful for exploratory studies 5. Involves larger number of subjects.
5. Involves fewer number of
6. Long follow-up period often
subjects
needed, involving delayed results
6. Yields relatively quick results 7. Inappropriate when the disease or


7. Suitable for the study of rare
exposure under investigation is
diseases.
rare.
8. Generally yields only estimate8. Yields incidence rates, RR as well
of RR
as AR (odds ratio)
9. Cannot yield information about9. Can yield information about more
diseases other than that
than one disease outcome.
selected for study
10. Expensive
0. Relatively inexpensive
11. less bias
1. Chances of bias are more


f)
Correct regarding management of
sewage -

36.
Active sludge processing is no longer recommended
37.
Sludge degradation involves both aerobic and
anaerobic decomposition
38.
Treated sludge is released into river water
39.
The strength of sewage depends on biological oxygen
demand
40.
None
Correct Answer - C:D
Ans. is 'c' i.e., Treated sludge is released into river water; &
'd' i.e., The strength of sewage depends on biological oxygen

demand [Ref: Park's 2e* p. 799-802]
Sewage is waste water from community, containing solid and liquid
excreta. It contains 99.9% water and 0.1% solids (organic and
inorganic).
'Dry weather flow' is the average amount of sewage that flows in
sewage system in 24 hours.
The segregation of excreta by imposing a barrier is called
"Sanitation barrier".
BOD value ranges from about 1 mg per litre for natural waters to
about 300 mg per litre for untreated domestic sewage.
If the BOD is 300 mg/I and above, sewage is said to be strong; if it is
100 mg/I, it is said to be weak."
There are following methods of disposal of effluent?
f) Disposal by dilution : Disposal into water courses such as rivers
and streams is called disposal by dilution.

37. Disposal on land : If suitable land is available the effluent can
be used for irrigation purposes (e.g. Okhla sewage treatment
plan in Delhi).


106. Post-exposure prophylaxis is given in -
f) Rabies
g) HBV
h) Influenza
i) Rubella
j) Measles
Correct Answer - A:B:E
Ans. is 'a' i.e., Rabies; 'b' i.e., HBV; & `e' i.e., Measles [Ref Park's
22"die p. 149, 278]
Post-exposure prophylaxis refers to prophylactic measure taken
after exposure to a pathogen, in order to prevent infection by
the pathogen and development of disease.
Post-exposure immunization
Here prophylactic vaccination is given after exposure. Post-
exposure immunization is given for varicella (chicken pox), rabies,
hepatitis-B, measles, tetanus, and meningococcal meningitis.
Post-exposure chemoprophylaxis
Here drugs are used for post-exposure chemoprophylaxis.
It is used in HIV, Herpes, diphtheria, and meningococcal meningitis.


107. True about 90:90:90 strategy -
f) For HIV treatment
g) Targets for 2025
h) 90% of people with HIV infection will receive treatment
i) 90% of people with HIV infection will know HIV status
j) 90% of people with HIV infection will be prevented from
TB infection
Correct Answer - A:C:D
Ans. is 'a' i.e., For HIV treatment; 'c' i.e., 90% people with HIV
infection will receive treatment; & 'd' i.e., 90% people with
HIV infection will know HIV status [Ref: www.unaids.org]

In Dec. 2013, the UNAIDS programme Coordinating Board called on
UNAIDS to support country -and region -led efforts to establish new
targets for HIV treatment scale-up beyond 2015. In response,
stakeholder consultations on new targets have been held in all
regions of the world. At the global level, stakeholders assembled in
a variety of thematic consultations focused on civil society,
laboratory medicine, paediatric HIV treatment, adolescents and
other key issues.
Powerful momentum is now building towards a new narrative on HIV
treatment and a new, final, ambitious, but achievable target:
By 2020, 90% of all people living with HIV will know their HIV status.
By 2020, 90% of all people with diagnosed HIV infection will receive
sustained antiretroviral therapy.
By 2020, 90% of all people receiving antiretroviral therapy will have
viral suppression.


f)
Biodegradable waste products,
disposing in which of the colour code of
the bags-

40.
Blue
41.
Black
42.
Green
43.
Yellow
44.
None
Correct Answer - D
Ans. is 'd' i.e., Yellow


f)
True about National Programme For
Control of Blindness is ?

41.
Started in 1962
42.
Apex National institute is located in AIIMS Delhi
43.
Not to involve NGOS in the programme
44.
Emphasis towards eye camp approach
45.
Medical colleges as Tertiary centers
Correct Answer - B:E
Ans. is 'b' i.e., Apex National institute is located in AIIMS Delhi;
f) 'e' i.e., Medical colleges as Tertiary centers [Refi Park's
24th/e
p. 458, 459]

NPCB was launched in 1976. India was the first country to launch a
national level programme for blindness.
Its objective was to reduce the prevalence of eye diseases in
general and the prevalence of blindness from 1.40% to 0.3%
by 2000 AD.
It is a '100% centrally sponsored Programme'. The Apex centre
(National Eye Institute) is Dr. Rajendra Prasad Centre for
Ophthalmic Sciences, New Delhi, AIIMS.
'World Bank' provides assistance to NPCB for control of cataract,
and also for assessment and aiding for development of funds. To
strengthen participation of Voluntary Organizations in the programme
and to earmark geographic areas to NGOs and Government
Hospitals to avoid duplication of effort and improve the performance
of Government Units like Medical Colleges, District Hospitals, Sub
Divisional Hospitals, community Health Centres,


Primary Health Centres etc.


f)
True about WHO classification of Dengue
fever ?

43.
Dengue hemorrhagic fever I and 2 are Dengue shock
syndrome
44.
Dengue hemorrhagic fever-1 is characterized by platelet
count <150000 cells/Cu mm
45.
Dengue hemorrhagic fever-2 is characterized by
Hematocrit increase >20%
46.
Dengue hemorrhagic fever-4 is characterized by
Hematocrit increase > 20%
47.
Dengue hemorrhagic fever-4 is characterized by platelet
count < 100000/Cu mm
Correct Answer - C:D:E
Ans. is 'c' i.e., Dengue hemorrhagic fever-2 is characterized by
Hematocrite increase >20%; 'd' i.e., Dengue hemorrhagic fever-4
is characterized by Hematocrite increase > 20%; & 'e' i.e., Dengue
hemorrhage fever-4 is characterized by platelet count...

DHF II Above signs and symptoms plus some evidence of
spontaneous bleeding in skin or other organs (black tarry stools,
epistaxis, bleeding from gums, etc) and abdominal pain,
Thrombocytopenia : Platelet count <100, 000/cu.mm.
Haematorcit rise 20% or more
DHF IV Signs as grade III plus profound shock with undetectable
blood pressure or puls Thrombocytopenia: Platelet count <
100,000/cu. Mm. Haemotocrit rise more than 20%.
DHF III and IV are Dengue Shock Syndrome


111. True about influenza epidemiology -
f) It shows cyclic trend
g) Pandemic is caused by influenza-B virus
h) Major reservoir is human
i) Epidemics occur every 2-3 years
j) Pandemic influenza is defined as 25 suspect cases of H1N1
Correct Answer - A:D:E
Ans. is 'a' i.e., It shows cyclic trend; 'd' i.e., Epidemics occur
every 2-3 years; & 'e' i.e., Pandemic influenza is defined as
25 Suspect cases of HiN, [Ref: Park's 24th/e p. 163-164]

Influenza virus a RNA virus, belongs to orthomyxovirus.
There are three viral subtypes : Type A (causes all pandemics and
most epidemics); type B; and type C (not circulating currently).
Currently the influenza viruses circulating in the world are : H, N, of
type A (causes swine flu); H, N, of type A; H3 N, of type A ; 1-1, Ni
of type A (causes bird flu or avian influenza); H7 N, of type A
(caused epidemic of avian influenza in China in 2013); and type B.
Influenza shows cyclic trend with epidemic occurring every 2-3 years
in case of influenza - A and every 4-7 years in case of influenza-B.
Pandemics are caused by only influenza - A every 10-15 years.
Influenza affects all ages and both sexes.
Source of infection of influenza is a clinical case or subclinical case.
Major reservoir of influenza virus exists in animals and birds.
Incubation period is 18-72 hours. Most of the infections are
subclinical. Clinical cases present with cough, fever, myalgia and
headache.


44. True about distribution of prevalence of
goitre in school age children ?
f) >5% defined area as endemic
g) 5% - 19.9% is defined as moderate iodine deficiency
h) 20% is defined as severe iodine deficiency
i) 20-30% is defined as moderate iodine deficiency
j) None
Correct Answer - A:D
Ans. is 'a' i.e., >5% defined area as endemic; & 'd' i.e., 20-30%
is defined as moderate iodine deficinecy [Ref Park 22"/e p.
578; OP Ghai p. 484]

Total goitre rate : Percentage of children aged 6-11 with palpable or
visible goitre. This is an indicator of iodine deficiency, which
causes brain damage and mental retardation.
Goitre are classified as Not visible, palpable and visible as shown
below.
The term 'endemic goitre' refers to a total goitre rate of greater than
5 percent in a given community.


113. Screening is a type of ?
45.
Primordial prevention
46.
Secondary prevention
47.
Primary prevention
48.
Tertiary prevention
49.
None
Correct Answer - B
Ans. is b i.e., Secondary prevention


f)
For treatment of extended drug
resistance which of the following drugs
are used EXCEPT?

f) Rifampicin
g) INH
h) Moxifloxacin
i) Capreomycin
j) Clofazimine
Correct Answer - A
Ans. is 'A' Rifampicin[Ref Park 24th/e p.199]
MDR-TB is defined as resistance to at least both INH and rifampicin.
Previously it was classified as Category IV under DOTS (DOTS-
PLUS).
The treatment is given in two phases, the intestive phase (IP) and
the continuation phase (CP). The total duration of treatment for
regimen for MDR-TB is 24-27 months, depending on the IP
duration
Regimen is :-
. Intensive phase (6-12 months) : Seven drugs : Capreomycin, PAS,
moxifloxacin, high dose INH, clofazimine, Linezolid, amoxyclay.
f) Continuation phase (18 months) : Six drugs : PAS,
moxifloxacin, high dose INH, clofazimine, linezolid, amoxyclay.


115. True about BCG vaccination ?
f) Usually given at birth
g) Can be given in pregnancy
h) Uses live attenuated vaccine
i) Immunodeficiency is a contraindication
j) Causes tuberculin test to become NEGATIVE
Correct Answer - A:C
Ans. is 'a' i.e, Usually given at birth; & 'c' i.e., Uses
live attenuated vaccine [Ref Park 24thie p. 194-195].

The vaccine is given by intradermal route, just above the insertion of
deltoid (usually left side). Dose of vaccine is 0.1 ml for all age, with
a strength of 0.1mg in 0.1m1. Vaccine is usually given at birth or at
6 weeks of age simultaneous with DPT and Polio.
Duration of protection is around 15-20 years. Efficacy of protection
varies for different diseases : for pulmonary tuberculosis it is zero,
for severe forms of TB it is 0-80% (average 50%) and for leprosy it
is 20-40% (note : BCG vaccination also gives some protection
against leprosy).
Storage - BCG vaccines are stable for several weeks at ambient
temprature in tropical climate, and for upto 1 year. If kept away
from direct light and stored in a cool environment. The vaccine
must be protected from exposure to light during storage (wrapped
up in double layer of red or black cloth.


116. COMBINED vaccines are available for
f) Hepatitis B
g) TAB
h) Pneumococcal vaccine
i) Typhoid
j) HPV
Correct Answer - A:B:D
Ans. is 'a' i.e., Hepatitis B; 'b' i.e., TAB; & 'd' i.e., Typhoid
[Ref Park's 23'/e p. 110; www.omicsonline.org]

If more than one kind of immunizing agent is included in the vaccine,
it is called a mixed or combined vaccine.
The aim of combined vaccines is to simplify administration, reduce
costs, minimize the number of contacts of the patient with the
health system, reducing the storage cost, improving timelines of
vaccination, and facilitating the addition of new vaccine into
immunization programme.
The following are some of the well-known combination:
DPT (Diphtheria-pertussis-tetanus)
DT (Diphtheria-tetanus)
DP (Diphtheria-pertussis)
DPT and typhoid vaccine
MMR (Measles, mumps and rubella)
DPTP (DPT plus inactivated polio)
Hepatitis A, and B
Hepatitis A, and typhoid.
DTwP (Diphtheria, tetanus, whole-cell pertussis)


.
True about Intrauterine contraception
device is?

a) Causes inhibition of ovulation
b) Copper-T is second generation IUD
c) Best time of insertion is Immediately after menstruation
d) Bleeding is common
e) Expulsion is more common with multiload-375
Correct Answer - B:C:D
Ans. is 'b' i.e., Copper-T is second generation IUD; 'c' i.e., Best time of insertion is
Immdiately after mensturation; & 'd' i.e., Bleeding is common.
First generation
These are non-medicated and inert IUDs. Examples are Lippes loop and Grafenberg's
ring. Second generation
These are medicated and bio-active IUDs. Metallic ions (Copper) are added to the
device. Examples are Cu T, CuT200, CuT-200B, CuT-200C, Nova-T, Multiload 375,
Multiload 250, Cut-380A.
Third generation
These are also medicated and bioactive IUDs. These are hormone releasing
IUDs e.g. progestasert and LNG-20 (Mirena).
Life span of different IUDs is different. CuT-380A has longest life span of 10 years.
Life span of different IUDs in descending order : CuT-380A (10 years) > Nova-T,
Multiload-375 (5 years) and LNG-20/Mirena (5 years) > CuT - 200B (4 yars) > CuT - 200
(3 years) > progestasert (1 years).


f)
Objectives of National Health Policy-2017

-
51.
Reduce IMR to 28 by 2019
52.
Reduce MMR to 100 by 2020
53.
Reduce Under five mortality to 20 by 2020
54.
Reduce Neonatal mertality to 15 by 2020
55.
Reduce TFR to 2.1 by 2025
Correct Answer - A:B:E
Ans. is 'a' i.e.,Reduce IMR to 28 by 2019; 'b' i.e., Reduce MMR
to 100 by 2020; & 'e' i.e.,Reduce TFR to 2.1 by 2025 [Ref
National
Health Policy 2017 Document-Ministry of Health &
Fantily Welfare]
National Health Policy 2017



f)
A lady on combined OCP forgot to take 3
consecutive pills in the first weak of pill
cycle. What should be done?

(E)
Has to take 3 pills immediately
(F)
Should take next pill as per schedule
(G)
Has to shift to IUCD
(H)
Has to use barrier method for 7 days
(I) Use emergency contraceptives
Correct Answer - B:D:E
Ans. is 'b' i.e., Should take next pill as per schedule; 'd' i.e.,
Has to use barrier method for 7 days; & 'e' i.e., Use emergency
contraceptives [Ref: www.enpowerhealth.com]



120. Vocal cord is supplied by -
52.
Internal laryngeal nerve
53.
Superior laryngeal nerve
54.
Recurrent laryngeal nerve
55.
Vagus nerve
56.
Glossopharyngeal nerve
Correct Answer - A:B:C:D
Answer- (A) Internal laryngeal nerve (B) Superior laryngeal
nerve (C) Recurrent laryngeal nerve (D) Vagus nerve

The main cranial nerve innervating the larynx is the vagus nerve via
its branches; superior laryngeal nerve (SLN) and
Recurrent laryngeal nerve (RLN).
Above the vocal cords the sensory innervation of larynx is via
internal laryngeal nerve.


f)
In bilateral recurrent laryngeal nerve
paralysis, which of the following is/are is
seen -

. Paramedian vocal cord
. Dyspnea is seen
. Stridor is seen
. No effect on voice
. Tracheostomy may be required
Correct Answer - A:B:C:D:E
Answer- (A) Paramedian vocal cord (B) Dyspnea is seen
f) Stridor is seen (D) No effect on voice (E) Tracheostomy
may be required


Bilateral RLN paralysis (Bilateral abductor paralysis)-
This is the most dangerous paralysis as both the cords are in
median or paramedian position ,obstructing the airway.
There is dyspnea and stridor.
Treatment
Tracheostomy
Lateralization of cord
Kashima operation


f)
True about tubercular otitis media are all
except?

(F)
Spreads through eustachian tube
(G)
Causes painless ear discharge
(H)
May cause multiple perforations
(I) Usually affects both ears
(J) None
Correct Answer - D
Answer- D. Usually affects both ears
Tuberculosis of middle ear is a comparatively rare entity usually
seen in association with or secondary to pulmonary
tuberculosis, infection reaches the middle ear through
eustachian tube.
Clinical features
Generally, tuberculosis of middle ear is unilateral.
It is characterized by painless otorrhoea which fails to respond to the
usual antimicrobial treatment. There is painless watery otorrhea.
Single or multiple perforation of tympanic membrane.


55. Clinical feature of facial palsy are all
except -
f) Loss of forehead wrinkling
g) Difficulty in closing eye
h) Loss of taste sensation from tongue
i) Paralysis of stapedius muscle
j) Loss of Gag reflexes
Correct Answer - A:B:C:D
Answer- (A) Loss of forehead wrinkling (B) Difficulty in
closing eye (C) Loss of taste sensation from tongue (D)
Paralysis of stapedius muscle

Weakness of the muscle of the facial expression and
eye closure

f) Absence of nasolabial fold
g)
Wide palpable fissure
h)
Epiphora
i) Drooping of angle of mouth
j) Loss of wrinkles of forehead
k)
The face sags and is drawn across to the opposite side on smiling.
l) Voluntary eye closure may not be possible and can produce damage


57. Contraindication of cochlear
implantation is/are -
f) Mondini deformity
g) Intracochlear ossification
h) Chronic suppurative otitis media
i) Agenesis of cochlear nerve
j) All
Correct Answer - C:D
Answer-(C) Chronic suppurative otitis media (D) Agenesis of
cochlear nerve
Absolute
58. Active middle ear infection: ASOM, CSOM, mastoiditis
59. Agenesis of cochlea and/or Cochlear nerve
60. Mental retardion:Patient cannot cooperate with speech training


125. True about schwartz's sign -
f) Seen in otic capsule
g) Indicates active disease
h) Surgery is the Treatment
i) Causes sensory - neural deafness
j) More common in pregnancy
Correct Answer - A:B:C:D:E
Answer- (A) Seen in otic capsule (B) Indicates active disease
f) Surgery is the Treatment (D) Causes sensory -
neural deafness (E) More common in pregnancy

Schwartz's sign, also known as Flemingo's flush sign or Rising sun
sign is believed to be associated with otopongiosis which is the
active phase of the disease, usually in pregnancy.
Schwartz's sign which refers to a reddish discoloration over the
Promontory seen beyond the intact tympanic membrane.
surgery rernains a therapeutic option.


.
Ectopia lentis is associated with all
except -

f) Homocystinuria
g) Weil - Marchesani syndrome
h) Marfan syndrome
i) Cockayne syndrome
j) Osteogenesis imperfecta
Correct Answer - D:E
Answer- (D) Cockayne syndrome (E) Osteogenesis imperfecta
More common:
Marfan syndrome
Homocystinuria
Weil-Marchesani syndrome
Sulfite oxidase deficiency
Hyperlysinemia


127. Feature of mycotic corneal ulcer -
f) Feathery margins
g) Non-sterile hypopyon
h) Satellite lesions
i) Ulcer serpens
j) Deep involvement
Correct Answer - A:B:C:E
Answer- (A) Feathery margins (B) Non-sterile hypopyon
(C) Satellite lesions (E) Deep involvement
Greyish -white dry looking ulcer with the elevated rolled out feathery
& hyphate margins.
Feathery finger like extension into surrounding stroma under intact
epithelium.
A sterile immune ring (yellow line) of Wesseley.
Multiple small satellite lesions.
Non-sterile (infected) hypopyon (Pseudohypopyon) containing
fungus.
Perforation is rare and corneal vascularization is conspicuously
absent


128. True about fundoscopic finding is/are
f) Lateral margin of optic disc is normally blurred
g) Optic disc is horizontally oval
h) Loss of spontaneaus retinal venous pulsation is seen
in increased intracranial tension
i) Blurring starts from nasal margin in increased
intracranial tension
j) None
Correct Answer - C:D
Answer- C,Loss of spontaneaus retinal... D,Blurring starts from
nasal...
Signs of papilloedema (increased intracranial tension)
Blurring or obscuration of disc margin (First sign). Blurring starts at
upper and lower nasal margins and extends around nasal side,
while temporal margin is last to involve.
Venous engorgment and venous congestion.
There is filling of physiological cup with gradual obliteration
ofphysiological cup.


129. Unilaterally dilated pupil is seen in
63.
Homer's syndrome
64.
Adie's pupil
65.
Argyll Roberston pupil
66.
61h Cranial nerve palsy
67.
3rd nerve palsy
Correct Answer - B:E
Answer- B,Adie's pupil E,3rd nerve palsy
Mydriatic eye drops (Atropine, scopolamine)
Postganglionic mydriasis (Adie's pupil)
Preganglionic mydriasis (3rd nerve palsy)
Acute angle closure gloucoma Ocular
Prosthesis
Trauma : postraumatic iridocyclitis
Physiological anisocor


130. Ocular findings in vitamin A deficiency
f) Parenchymatous conjunctival xerosis
g) X-2 is conjunctival xerosis
h) Earliest symptom is night blindness
i) Conjunctival xerosis is earliest ocular sign
j) All
Correct Answer - C:D
Answer- C,Earliest symptom is night blindness
D,Conjunctival xerosis is earliest ocular sign

The most characteristic and specific signs of vitamin A deficiency
are eye lesions.
Xerophthalmia (dry eye)
It has following sequential stages night blindness (earliest ocular
symptom), conjunctival xerosis (earliest ocular sign), Bitot's
spot, corneal xerosis and Keratomalacia with corneal ulcer.
Xerosis of conjunctiva refers to a condition where the conjunctiva
becomes dry and lusterless.


131. True about adenovirus conjunctivitis -
f) One of the most common cause of viral conjunctivitis
g) It is less contagious than other viral conjunctivitis
h) Pharyngoconjunctival fever is caused by 3 & 7 types
i) May cause hemorrhagic conjunctivitis
j) Conjunctival follicles
Correct Answer - A:C:D:E
Answer- A,One of the most common cause of viral
conjunctivitis C,Pharyngoconjunctival fever is caused by 3 &
7 types D,May cause hemorrhagic conjunctivitis
E,Conjunctival follicles

Viral conjunctivitis is most frequently caused by an adenovirus.
Pharyngoconjunctivial fever (PCF) is caused by adenovirus serovars
3,4 and 7.
The spread of this highly contagious disease is facilitated by the
ability of viral particles.
Prominent conjunctival hyperaemia and follicles.
Severe inflammation maybe associated with conjunctival
heamorrhages (usually petechial in adenoviral infection),
Chemosis,membranes (rare) and pseudomembranes.


132. True about discharge from eye
(F)
Epiphora is due excessive secretion of tear
(G)
Mucopurulent discharge in acute conjunctivitis
(H)
Mucopurulent discharge in acute iridocyclitis
(I) Watery discharge in acute congestive glaucoma
(J) None
Correct Answer - B:D
Answer- B,Mucopurulent discharge in acute
conjunctivitis D,Watery discharge in acute
congestive glaucoma

Acute conjuctivitis-
Discharge- Mucopurulent
Coloured halos- May be present
Acute iridocylitis-
Discharge- Watery
Coloured halos- Absent
Acute congestive glucoma-
Discharge- Watery
Coloured halos- Present
Obstruction to the outflow of normally secreted tears or due to
lacrimal pump failure --> Epiphora
Excessive secretion of tears hyperlacrimation


65. Which of the following is/are primary
glaucoma
f) Infantile glaucoma
g) Open angle glaucuma
h) Steroid induced glaucoma
i) Aphakic glaucoma
j) Phacogenic glaucoma
Correct Answer - A:B
Answer- A,Infantile glaucoma B,Open angle
glaucuma Primary adult glaucomas

Primary open angle glaucoma
Primary angle closure glaucoma
Primary mixed mechanism glaucoma
Primary congenital / developmental (without associated anomalies):-
Congenital, Infantile,Juvenile


134. Cause(s) of cystoid macular edema is/are
. Diabetes mellitus
. After cataract surgery
. Retinitis pigmentosa
. Rheugmatogenous retinal detachment
. Pilocarpine drop
Correct Answer - A:B:C
Answer- A,Diabetes mellitus B,After cataract surgery
C,Retinitis pigmentosa

Causes of macular edema are-
Metabolic alteration: - Diabetes, retinitis pigmentosa , Inherited
cystoid macular edema (CME).
Ischemia; - CRVO, Diabetic retinopathy, severe hypertensive
retinopathy, HELLP syndrome, vasculitis.
Mechanical force : - Vitreous traction on the macula.
Inflammation : - Intermediate uveitis, Post-operative CME, choroidal
inflammatory diseases.
Pharmacotoxicity: - Epinephrine (in Aphakia), Betaxolal,
Latanoprost.


f)
Restrictive lung disease differs from
obstructive lung disease by

67.
Decreased FVC
68.
Decreased FEV1
69.
Decreased TLC
70.
Decreased RV
71.
Decreased FEV1/FVC
Correct Answer - A:B:C:D
Answer- A,Decreased FVC B,Decreased FEV1
C,Decreased TLC D,Decreased RV

PET Result for Restrictive lung disease-
FEV1- Decreased
FVC- Decreased
FEV1/ FVC- Normal or increased
TLC- Decreased
DLCO- Decreased in intrinsic restrictive lung disease


136. Fifth cranial nerve palsy causes
f) Weakness of opening of mouth
g) Weakness of closure of mouth
h) Loss of corneal reflex
i) Loss of lacrimal reflex
j) Loss of taste sensation from anterior 2/3 of tongue
Correct Answer - A:B:C:D
Answer- A,Weakness of opening of mouth B,Weakness of
closure of mouth C,Loss of corneal reflex D,Loss of
lacrimal reflex

Opening of mouth is caused by both lateral pterygoid --> Supplied
by mandibular branch of trigeminal nerve.
68. One side injury to trigeminal nerve causes weakness of
opening of mouth and deviation of jaw to affected side.
. Both side palsy causes weakness of opening of mouth.
Corneal reflex and lacrimal reflex pathway involve trigeminal nerve.
Corneal reflex - Afferent limb is formed by ophthalmic nerve and
efferent limb is facial nerve.


f)
Patient came with complaints of
Polydipsia, hypercalciurea,
nephrolithiasis, metabolic alkalosis.
Possible cause is

69.
Bartters syndrome
70.
Gittlemans syndrome
71.
Addisons disease
72.
Chronic diuretic use
73.
None
Correct Answer - A
Answer- A. Bartters syndrome
Bartter syndrome is an autosomal recessive disorder caused by
mutation in gene coding for basolateral chloride channel (ClC-
kb). There is loss of sodium, chloride, potassium and calcium in
urine.
The major clinical findings are hyponatremia, hypokalemia, polyurea,
polydipsia, metabolic alkalosis, normal to low
BP, hypomagnesemia (only in some patients), hypochloremia,
hypercalciuria (causing nephrocalcinosis), and growth


f)
True about ECG findings of ventricular
premature beat is/are

70.
Increase R-R interval
71.
ST segment depression
72.
ST segment elevation
73.
T wave inversion
74.
Obsured P wave
Correct Answer - B:C:D:E
Answer- B,ST segment depression C,ST segment elevation D,T
wave inversion E,Obsured P wave ST-T wave

When QRS complex upight
S-T segment is depressed and convex upwards
T wave is inverted
When QRS complex downward
S-T segment is elevated and concave upwards
T wave is upright


f)
Patient comes with mild dyspnea. On
ECG monomorphic ventricular
tachycardia was found, which of the
following drug is to be used

71.
Adenosine
72.
Lignocaine
73.
Amiodarone
74.
Propranolol
75.
Procainamide
Correct Answer - B:C:D:E
Answer-
B,Lignocaine C,Amiodarone D,Propranolol
E,Procainamide Stable patient with monomorphic VT

f) If left ventricular function is
normal- IV procainamide or

IV amiodarone or

IV sotalol/propranol/ esmolol

Lidocaine may also be used

g) Impaired left ventricular function
IV amiodarone or lidocaine are preferred


72. Capnography helps to know the
following
f) Correct intubation
g) Pulmonary embolism
h) Adequate ventilation
i) Lung perfusion
j) Significant metabolic change
Correct Answer - A:B:C:D:E
Answer- A,Correct intubation B,Pulmonary
embolism C,Adequate ventilation D,Lung
perfusion E,Significant metabolic change

Conditions that affect ET CO2
Increased
Hypoventilation
Rebreathing
Malignant hyperthermia,
Neuroleptic malignant syndrome
Increased skeletal muscle activity (shivering
Hypermetabolism
Hyperthyroidism & thyroid storm
Decreased
Hyperventilation
Pulmonary embolism
Hypoperfusion, hypotension, hypovalemia, shock
Hypothermia


f)
Not true regarding
mucopolysccharidosis is/are

74.
They are lysosomal diseases
75.
All are autosomal dominant except Sanfilippo syndrome
76.
They have common skeletal feature: Dysostosis multiplex
77.
Hurler syndrome is due to deficiency of Iduronate sulfatase
78.
Morquio syndrome [IV] is due to deficiency of
Beta-galactosidase
Correct Answer - B:D
Answer- B,All are autosomal dominant except Sanfilippo
syndrome D,Hurler syndrome is due to deficiency of
Iduronate sulfatase

Mucopolysaccharidosis (MPS) represent a heterogenous group of
inheritable lysosomal storage diseases in which the accumulation
of undegraded glycans leads to progressive damage of affected
tissues.
Hurler (IH) syndrome- Alpha L-iduronidase
Corneal clouding, dysostosis multiplex, organomegaly; heart
disease, mental retardation
Sanfilippo syndrome- Autosomal recessive.
Morquio IV syndrome- Beta-galactosidase, Galactose-6- sulfate
sulfatase
Maroteaux-Lamy syndrome- N-acetylgalactosamine (Dysostosis
multiplex)


f)
Drugs used in bladder/urinary
incontinence

75.
Oxybutynin
76.
Tolterodine
77.
Trospium
78.
Neostigmine
79.
Demecarium
Correct Answer - A:B:C
Answer- A,Oxybutynin B,Tolterodine C,Trospium
Selective M3 antagonists- Oxybutynin, darifenacin, Tolterodine
Nonselective antagonist- Trospium, Propiverine
Tricyclic antidepressant- Imipramine


f)
Which of the following is/are included in
the management of stress incontinence

f) Botulinum toxin
g) Kelly procedure
h) Antichoinergic
i) Urethropexy
j) Imipramine
Correct Answer - B:C:D:E
Answer- B,Kelly procedure C,Antichoinergic
D,Urethropexy E,Imipramine

There are 4 types of treatment-
77. Behavior changes
78. Medicine
Anticholinergic medicines help relax the muscles of the bladder.
Antimuscarinic drugs block bladder contractions.
Imipramine, an antidepressant, helps relax bladder muscles.
f) Pelvic floor muscle training
g) Surgery
Anterior vaginal repair (anterior colporrhaphy or Kelly procedure)
helps restore weak and sagging vaginal walls.
Artificial urinary sphincter
Bulking injections make the area around the urethra thicker.
Burch urethropexy and Marshall-Marchetti-Krantz (MMK) procedure


(vii) Primaqunie sensitivity is seen in anemia
with following enzyme deficiency except
78.
Pyruvate dehydrogenase
79.
Hexokinase
80.
Glucose 6 phosphate dehydrogenase
81.
Glucose 6 phosphate
82.
None
Correct Answer - A:B:D
Answer- A,Pyruvate dehydrogenase B,Hexokinase D,Glucose
6 phosphate

It is active on pre and exo-erythrocytic stage. (Primaquine is the only
antimalarial which is active on exoerythrocytic stage). It acts by
interfering mitochondrial function.
Those with G-6-PD deficiency are highly sensitive anil hemolytic
anaemia can occur.
Therefore it should be avoided in pregnancy as fetus with G6PD
deficiency may develop hemolytic anemia.


145. Red color of urine is caused by
f) Aniline dye
g) Beet root ingestion
h) Rifampicin
i) Penol intake
j) Alkaptonuria
Correct Answer - A:B:C
Answer- A,Aniline dye B,Beet root ingestion
C,Rifampicin Hematuria
Porphyria
Serratia marcescens
Aniline dyes
Ingestion of blackberries
Ingestion of beetroot
Phenolphthaliene
Phenytoin
Rifampin (red brown orange)
Pyridium


f)
Solute induced diuresis is characterized
by -

. Polyuria
. Decreased urine osmolality
. Urine: Plasma osmolality > 0.7
. Osmotic clearance > 3 ml/min
. Urine output > 3L/day
Correct Answer - A:C:D:E
Answer- (A) Polyuria (C) Urine: Plasma osmolality > 0.7
.
Osmotic clearance > 3 ml/min (E) Urine output >
3L/day Urine-to Plasma osmolality ratio >0.7

Osmotic clearance >3 mL/min

Polyuria Urine osmolality (>300 mosmol) Solute
diuresis Glucose, mannitol,radiocontrast, urea(from high
proteinfeeding) medullary cystic diseases, resolving ATN,
or obstruction, diuretics.


147. Milk-alkali syndrome is associated with -
. High PTH
. Hypercalcemia
. Metabolic acidosis
. Elevated creatinine
. Hyperphosphatemia
Correct Answer - B:D:E
Answer- (B) Hypercalcemia (D) Elevated creatinine
(E) Hyperphosphatemia
The syndrome is characterized by -
. Hypercalcemia
. Hyperphosphatemia
. Metabolic alkalosis
. Metastatic calcification
. Progressive renal failure (inreased, BUN and creatinine)


148. Continuous murmur is seen is -
f) Patent ductus arteriosus
g) Tetrology of fallot
h) Pregnancy
i) Coarctation of aorta
j) Ventricular septal defect
Correct Answer - A:C:D
Answer- (A) Patent ductus arteriosus (C) Pregnancy
(D) Coarctation of aorta
Common Causes of continuous murmurs
Systemic arteriovenous fistula (congenital /acquired)
Coronary arteriovenous fistula
Anomalous origin of Left coronary artery from pulmonary artery
Communication between sinus of valsalva and right side of heart
(i.e.Ruptured sinus of valsalva into right side of heart)
Coarctation of Aorta : Continuous murmur in the back
Patent Ductus Arteriosus (PDA)
Surgically created shunts e.g. Blalock-Tausig shunt
Mammary souffle (pregnancy)


149. Early diastolic murmur is seen in which
condition(s) -
a) Mitral stenosis
b) Tricuspid stenosis
c) Aortic regurgitation
d) Pulmonary regurgitation
e) Atrial myxoma
Correct Answer - C:D
Answer- (C) Aortic regurgitation (D) Pulmonary regurgitation
Aortic regurgitation- The murmur is low intensity, high-pitched, best
heard over the left sternal border or over the right second
intercostal space.
An Austin Flint murmur is usually associated with significant aortic
regurgitation.
Pulmonary regurgitation- Pulmonary regurgitation is most commonly
due to pulmonary hypertension (Graham- Steell murmur)
Left anterior descending artery stenosis- This murmur, also known
as Dock's murmur.


150. Not true about kaposi sarcoma -
a) Caused by HHV-8
b) Classical form is associated with HIV
c) Is an angioproliferative disorder
d) Monocentric tumor
e) May involve GIT
Correct Answer - B:D
Answer- (B) Classical form is associated with HIV
(D) Monocentric tumor
KaPosi sarcoma is multicentric vascular tumor caused by Human
herpes virus-8 (HHV-8) also called Kaposi sarcoma associated
herpes virus (KSHV).
There are four forms of Kaposi sarcoma-
l) Classical form (European or Mediterranean KS)
There is no association with HIV. There are skin plaques and
nodules.
2) African form (Endemic form or Equatorial form)
There is no association with HIV. There is lymphadenopathy
3) Transplant associated (immunosuppression associated) KS
4) AIDS cssociated (Epidemic) KS
It is associated with HIV infecfion


151. Vascular changes of malignant
hypertension is are -
a) Hyaline arteriosclerosis
b) Necrotizing arteriolitis
c) Hyperplastic arterioscleroesis
d) Aortic dissection
e) Onion skinning
Correct Answer - B:C:D:E
Answer- (B) Necrotizing arteriolitis (C) Hyperplastic
arterioscleroesis (D) Aortic dissection (E) Onion
skinning Hypertension is associated with two forms-1.
Hyaline arteriolosclerosis-

It is characteristic of benign hypertension.
2. Hyperplastic arteriosclerosis
It is characteristic of malignant hypertension
Onion skinning
There is mucinous intimal thickening and fibrous intimal thickening.
There may be accompanied fibrinoid deposits with necrosis of the
vessels wall --> fibrinoid necrosis (or necrotizing arteriolitis).


152. All of the following statements about
Neurofibromatosis are true, Except:
a) Autosomal Recessive Inheritance
b) Cutaneous neurofibromas
c) Cataract
d) Scoliosis
e) None
Correct Answer - A
Answer- A. Autosomal Recessive Inheritance
Neurofibromatosis is inherited as an autosomal dominant condition.
Peripheral Neurofibromatosis (Von Recklinghausen's syndrome)
Most prevalent type (90%)
Diagnostic Criteria for NF I
Diagnosed when any two of the following are present
. Six more cafe-au-loit macules over 5 mm in greatest diameter in
prepubertal individuals and over 15 mm in greatest diameter in
post-pubertal individuals.
. Axillary or inguinal freckling
. Two or more iris Lisch nodules
. Two or more neurofibromas or one plexiform neurofibroma
. A distinctive osseous lesion such as sphenoid dysplasia or cortical
thinning of long bone, with or without pseudoarthrosis. 6. Optic
gliomas.
. A first degree relative with NFI whose diagnosis was based on the
aforementioned criteria.
Scoliosis is the most common orthopaedic manifestation NFl.




153. True about primary sclerosing
cholangitis:
a) Involves only intrahepatic bile duct, not extrahepatic bile duct
b) Associated with Inflammatory bowel disease
c) Causes macronodular cirrhosis
d) Periductal fibrosis of smaller bile ducts
e) None
Correct Answer - B:D
Answer- (B) Associated with Inflammatory bowel disease
(D) Periductal fibrosis of smaller bile ducts
PSC is characterized by lnflammation and obliterative fibrosis of
lntrahepatic and extrahepatic bile ducts wlth dilation of
preserved segments.
Inflammatory bowel disease , particularly ulcerative colitis, coexists
in approximately 70% of individuals with PSC.
Primary sclerosing cholangltis causes micronodular cirrhosis.
Following changes are seen-fibrosing cholangitis, periductal fibrosis,
dilation of intervening bile ducts and cholestatis with full blown picture
of biliary cirrhosis"


154. Increased portal venous pressure is seen
in
a) Budd chairi syndrome
b) Cirrhosis
c) Portal venous thrombosis
d) IVC obstruction
e) None
Correct Answer - A:B:C:D
Answer- A,Budd chairi syndrome B,Cirrhosis C,Portal
venous thrombosis D,IVC obstruction

MC cause of portal hypertension : Cirrhosis.
Portal hypertension results in splenomegaly with enlarged, tortuous,
and even aneurysmal splenic vessels.
Most bleeding episodes occur during the first I to 2 years after
identification ofvarices.
Colour Doppler is the investigation of choice for evaluation of PHT.
Splenic pulp pressure gives a measure of the portal vein pressure.
Posthepatic
Budd-Chiari syndrome
Inferiorvena caval webs
Hepatic-
Sinusoidal
Cirrhosis
Alcoholic hepatitis


155. Which of the following suture materials
are not cleared by proteolytic enzymes
a) Catgut
b) Vicryl
c) Nylon
d) Polyglactin
e) Polyamide
Correct Answer - B:C:D:E
Anwer- B,Vicryl C,Nylon D,Polyglactin E,Polyamide
A) Natural
Silk

Linen

Cotton

B) Synthetic
Nylon

Polypropylene

Polybutest

Braided polysters


156. Along with surgical treatment of
glioblastoma multiforme, following drug
is approved for its treatment

a) Cisplatin
b) Methotrexate
c) Cytarabin
d) Paclitaxel
e) Temozolomide
Correct Answer - A:E
Answer- A,Cisplatin E,Temozolomide
Carmustine (BCNU)
Cisplatin
Bevacizumab (avastin)


157. Which of the following is not true
a) Reef knot is stronger than Granny knot
b) Granny knot is stronger than Reef knot
c) Reef knot is applied for tissue under pressure
d) Reef knot is stronger than Surgeon's knot
e) Reef knot is better than slip knot for bowel anastomosis
Correct Answer - B:D:E
Answer- B,Granny knot is stronger than Reef knot D,Reef
knot is stronger than Surgeon's knot E,Reef knot is better
than slip knot for bowel anastomosis

Square knot is formed by wrapping the suture around the needle
holder once in opposite direction between ties.
Usually 3 ties are recommended.
Square (Reef) knot is more stable than Granny knot - has less
tendency to slip wher subjected increased pressure.
It may be used to tie Surgical gut, virgin silk, surgical cotton and
surgical stainless steel.


158. True about chest wall tumor is /are
a) Lipoma is the most common tumor
b) Fibrous dysplasia is a common skeletal sarcoma
c) Desmoid tumor is usually not malignant
d) Chondrosarcoma is the most common malignant tumor
e) Treated by surgical resection with wide surgical margins
Correct Answer - B:D:E
Answer- B,Fibrous dysplasia... D,Chondrosarcoma is the
most... E,Treated by surgical resection...
More than half of the cest wall tumors are malignant.
The most common benign chest wall tumors are:
Osteochondroma
Chondroma
Fibrous dysplasia
Chondrosarcomas are the most common primary chest wall
sarcoma and arise from the anterior tract of ribs and less
commonly from the sternum, scapula, or clavicle.
The most common soft-tissue primary malignant tumors are
fibrosarcomas.
Most chest wall tumors are treated with surgical resection and
reconstruction.


159. True regarding management of axillary
lymph nodes in breast carcinoma
a) All lymph-nodes are dissected
b) Lymph node dissection is done in modified radical mastectomy
c) Can be explored through the extended incision of breast
d) Complete dissection is required if 2 sentinel lymph nodes
are positive
e) None
Correct Answer - B:C
Answer- B,Lymph node dissection is done in modified
radical mastectomy C,Can be explored through the extended
incision of breast

All lymph nodes are not dissected in breast carcinoma, specially in
stage I & II
Lymph nodes are dissected in modified radical mastectomy
Axilla can be explored through the vertical incision taken for breast
Complete dissection is not required of 1 or 2 sentinel lymph nodes
are positive in stage I & II


160. Level VI lymph nodes in neck -
a) Anterior compartment
b) Prelaryngeal
c) Submental
d) Nodes of upper mediastinum
e) Jugular nodes
Correct Answer - A:B
Answer- (A) Anterior compartment (B) Prelaryngeal
Level VI
Pretracheal
Paratracheal
Prelaryngeal


161. True about Abdominal compartment is
/are -
a) Intraabdominal pressure > 15 mm Hg
b) Increased intracranial pressure
c) Decreased cardiac output
d) Hypoventilation
e) Renal failure and oliguria
Correct Answer - B:C:D:E
Answer- (B) Increased intracranial pressure (C) Decreased
cardiac output (D) Hypoventilation (E) Renal failure and oliguria

ACS is defined as an increase in intra- abdominal pressure (IAP)
more than 20mm Hg associated with new organ failure/
dysfunction.
Physiological Consequences of Increased Infra-abdominal
Pressure-
Decreased-
Cardiac Output
Central Venous Return
Visceral blood flow
Renal blood flow
Glomerular filteration
Increased
Cardiac rate
Pulmonary capillary wedge pressure
Peak inspiratory pressure
Intrapleural pressure
Hypoventilation and alteration of ventilation/perfusion distribution


lead to hypoxemia and hypercapnia.


162. Most common site of splenic implant in
abdomen -
a) Stomach
b) Tail of pancreas
c) Peritoneal cavity
d) Suprarenal gland
e) None
Correct Answer - C
Answer- C. Peritoneal cavity
Hetertopic splenic implant (splenosis) may occur in the peritoneal
cavity, after splenic trauma or splenectomy.
Splenosis is autotransplantation of splenic tissue after disruption of
splenic capsule by trauma or surgery.
Most common areas of implantalion are peritoneum, omentum and
mesentry.


163. Symptoms of mesenteric ischemia is /are

-
a) Severe abdominal pain
b) Degree of tenderness is proportionate to abdominal pain
c) Nausea and vomiting
d) Diarrhea
e) Absent bowel sound
Correct Answer - A:C:D:E
Answer- (A) Severe abdominal pain (C) Nausea and vomiting
(D) Diarrhea (E) Absent bowel sound
Ischaemia affecting the small intestine referred to as mesentric
ischaemia.
Intestinal ischemia can be of two types-
Acute mesenteric ischemia (the common type)
Chronic mesenteric ischemia C/F
Severe abdominal pain, out ofproportion to the degree of tenderness
on exarnination, is the hallmark ofacute mesenteric ischemia.
Associated symptoms can include nausea, vomiting, and diarrhea.


164. Treatment of acute pancreatitis
include(s) -
a) Intravenous fluid
b) Early ERCP
c) NSAIDs
d) Nasogastric tube
e) Octeride
Correct Answer - A:C:D
Answer- (A) Intravenous fluid (C) NSAIDs (D) Nasogastric tube
Mainly conservative - fluid resuscitation, analgesics, antibiotics and
anti-emetics
No role for TPN(to rest the pancreas); for nutritional support ?
enteral (nasogastric) feeding
In patients with severe acute gall stone pancreatitis and signs of
ongoing biliary obstruction and cholangitis, an urgent ERCP
should be performed
In patients with cholangitis ? sphincterotomy or a biliary stent
Indications for surgery: deterioration despite conservative therapy,
pancreatic abscess/necrosis.


165. Bilateral Hilar lymphadenopathy is seen
in -
a) Berylliosis
b) Silicosis
c) Amyloidosis
d) Occupational diseases
e) All
Correct Answer - A:B:D
Answer- (A) Berylliosis (B) Silicosis (D) Occupational diseases
Occupational: Silicosis, coal worker, pneumoconiosis, Berylliosis
TB (unilateral is more common)
Castleman's disease
Angioimmunoblastic lymphadenopathy
Phenytoin therapy


166. True about lipoma is/are -
a) Most common benign tumor of adults
b) Multiple lipoma can occur and are called as Dercum's disease
c) Intramuscular lipoma may cause pain
d) Rarely may occur in brain
e) May show slip sign
Correct Answer - A:B:C:E
Answer- A,Most common benign tumor of adults
B,Multiple lipoma can occur and are called as Dercum's

disease C,Intramuscular lipoma may cause pain E,May
show slip sign

Lipoma is a benign tumor of mature adipose tissue,
Lipoma is the most common benign tumor in adults.
Common sites are subcutaneous tissue over the trunk, nape of the
neck and limbs.
Slip sign is positive : Lipoma tends to slip away from the examining
finger on gentle pressure.
Presence of multiple lipomas is known as lipomatosis and multiple
lipomas are called Dercum's disease.
Intramuscular lipoma may interfere with muscle function and causes
pain on muscle action.


167. Macrocephaly is seen in ?
a) Soto's syndrome
b) Gorlin Syndrome
c) Achondroplasia
d) Maternal diabetes
e) Struge-Weber syndrome
Correct Answer - A:B:C:E
Ans. is 'a' i.e., Soto's syndrome; 'b' i.e., Gorlin Syndrome;
'c' i.e., Achondroplasia; &'e' i.e., Struge-Weber syndrome
Syndromes:

Fragile-X syndrome
Neuro-cutaneous syndromes
Tuberous sclerosis
Sturge-Weber
Increased CSF:
Hydrocephalus,
Choroid plexus papilloma.
Bone disease :
Achondroplasia
Osteogenesis imperfecta
OsteoPetrosis.
Others :
AV malformation
Intracranial haemorrhage
Thalassemia major
Hypervitaminosis-A
Lead poisoning

Pseudotumor cerebri
Galactosemia
Canavan's leukodystrophy.
Overgrowth syndromes :
Soto syndrome (Cerebral gigantism)
Weaver syndrome
Simpson-Golabi-Behmel syndrome (bulldog syndrome)
Macrocephaly -Capillary malformation (M-CMTC) syndrome.
Neuro-cardio facial-cutaneous syndromes:
Noonan syndrome
Costello syndrome
Gorlin syndrome (Basal Cell Nevus syndrome)
Cardio-facio-cutaneous syndrome.
Fragile-x syndrome & leukodystrophies (Alexander disease,
Canavan disease).


168. Large Skull size of newborn is seen in ?
a) Fetal alcohol syndrome
b) Gestational diabetes
c) Turner's syndrome
d) Canavan's leukodystrophy
e) Neurofibromatosis
Correct Answer - D:E
Ans. is'd'i.e., Canavan's leukodystrophy; & 'e'i.e.,
Neurofibromatosis
Canavan disease:
Autosomal-recessive neurological disorder associated with
macrocephaly and spongiform degeneration of brain.
There is either a lack of development or rapid regression of
psychomotor function, loss of sight and optic atrophy, lethargy,
difficulty in sucking, irritability, reduced motor activity, hypotonia
followed by spasticity of the limbs with corticospinal signs, and
an enlarged head (macrocephaly).


169. Clinical features of B-cell ALL in children
include:
a) Fever
b) Intrauterine death
c) Shock
d) Rigor mortis
e) Mediastinal enlargement
Correct Answer - A:B:C:D
Ans. is'a'i.e., Fever;'b'i.e., Intrauterine death'c'i.e.,
Shock &'d'i.e., Rigor mortis

Clinical manifestations of ALL :
Symptoms related to depression of normal marrow function. :
Anemia
Neutropenia
Fatigue, pallor
Infection, intermittent lever
Thrombocytopenia
Bleeding, petechiae, ecchymoses, epistaxis.
Bone pain and tenderness
Generalized lymphadenopathy, splenomegaly and hepatomegaly
Symptoms related to compression of large mediastinal vessels or
airway
CNS manifestation :
Headache, vomiting, nerve palsies.
Serious infections may cause Septic shock and life threatening
bleeding.

Due to neutropenia, there maybe infectious diarrhea.


170. True about heart sounds/murmur in atrial
septal defect -
a) Wide split S,
b) Early diastolic murmur
c) Loud shunt murmur
d) Delayed P2
e) Attenuation of S1
Correct Answer - A:D
Ans. is'a'i.e., Wide split S2 & 'd'i.e., Delayed
P2 Clinical manifestations of ASD
Patients with ASD are generally asymptomatic.
Mild effort intolerance and respiratory tract infection may occur.
CHF is rare.
Physical examination
Parasternal impulse
Systolic thrill at 2nd left interspace.
Accentuation of S, due to loud tricuspid component.
Wide split and fixed S2.
Ejection systolic murmur at the second and third left interspaces.
Delayed diastolic murmur at the lower left sternal border.
ASD with mitral stenosis Lutembacher syndrome.
Chest x-ray in ASD
Mild to moderate cardiomegaly artery segment.
Prominent pulmonary
Right atrial and right ventricular enlargement.
Relatively small aortic shadow

Plethoric lung fields.


171. True about murmur(s) in Patent Ductus
arteriosus -
a) Delayed diastolic murmur
b) Continuous murmur
c) Reverse splitting of S2
d) Ejection systolic murmur
e) Mid diastolic murmur
Correct Answer - A:B:C:D
Ans. is 'a' i.e., Delayed diastolic murmur; 'b' i.e.,
Continuous murmur; 'c' i.e., Reverse splitting of S2 ; &'d'
i.e., Ejection systolic murmur

Pressure gradient between aorta and pulmonary artery is maintained
throughout the cardiac cycle (during systole and
diastole) Continuous murmur, i.e., murmur starts in systole after
S1 , and reaches a peak at S2. It then diminishes and audible only
a part ofdiastole.
Larger blood volume passes through pulmonary circulation (blood
from right side of heart plus some blood from aorta)
Pulmonary plethora which may cause pulmonary hypertension.
Increased flow after passing through lung reaches the left atrium
and causes volume overload Left atrial dilatation and
hypertrophy.
Increased blood volume passes from left atrium to left ventricle
through mitral valve, i.e., increased flow through mitral
valve Accentuation of S1 and delayed diastolic murmur.
Left ventricle receives larger amount of blood that results in volume
overload Left ventricle enlargement.

Extra volume passes through aortic area cause delayed closure of
aortic valve which may close even after pulmonary valve (normal
pulmonary valves close after aortic valves). Paradoxical splitting
of S2, i.e., A, occurs after P.
Large left ventricular volume ejected into the aorta results in
dilatation of the ascending aorta Aortic ejection click.
Large volume of blood passes through normal aortic valve Aortic
ejection systolic murmur


172. Difference in Murmurs of PDA and ASD
is/are -
a) Delayed P2 in PDA
b) Wide split of S2 in PDA
c) Accentuation of S1 in ASD
d) Continuous murmur in PDA
e) Delayed diastolic murmur in ASD
Correct Answer - D
Ans. is'd'i.e., Continuous murmur in PDA
Delayed P2 and wide split S2 are feature of ASD (not PDA).
There is continuous murmur in PDA.


173. Radiological features of TOF is/are -
a) Cardiomegaly
b) Boot shaped heart
c) Right sided aortic arch
d) Pulmonary Plethora
e) Coeur an sabot
Correct Answer - B:C:E
Ans. is 'b' i.e., Boot shaped heart; 'c' i.e., Right sided
aortic arch; &'e' i.e., Coeur an sabot

Radiological features of TOF
Boot shaped heart (Coeur an sabot)
Normal heart size Oligaemic lung
fields
Right aortic arch (in 25%)


174. True about RETT Syndrome ?
a) Macrocephaly
b) Cardiac arrhythmia
c) Seizures
d) Mental retardation
e) Autistic behaviour
Correct Answer - B:C:D:E
Ans. is'b'i.e., Cardiac arrhythmia,'c'i.e., Seizures,'d'i.e.,
Mental retardation &'e'i.e., Autistic behaviour Rett's
Syndrome

This is the characteristic features, that they begin to loose their
acquired skills, e.g., cognitive and head growth is normal
during early period after which there is an arrest of growth.
Acquired microcephaly
Most children develop peculiar sighing respirations with intermittent
periods of apnea that may be associated with cyanosis Breath
holding spells.
Autistic behaviour Impaired social interaction, language and
communication.
Generalized tonic-clonic convulsions occur in the majority.
Feeding disorder and poor weight gain


175. True about applying a plaster is are ?
a) Taken out from water once bubbles start coming
b) Taken out from water once bubbles stop coming out
c) Setting is delayed in cold water
d) Cotton padding is overlapped by one third
e) Molding/smoothing of plaster is done by fingers
Correct Answer - B:C:D
Ans. is'b' i.e., Taken out from water once bubbles stop
coming out; 'c' i.e., Setting is delayed in cold water; 'd' i.e.,
Cotton padding is overlapped by one third

For securing, each turn is overlapped by one third to one half in
order to secure layers.
If a bandage is immersed in cold water the initial set will be delayed
and thus "working time" lengthened. However, if a very rapid is
required soaking the bandage in warm water will accelerate the
rate of reaction.
Moulding of the bandages to the contours of the limb should be done
by constant smoothing with the palms (not by fingers) of the wet
hands.


2.
Radiological features of Perthe's disease
is/are ?

3. Decreased medial joint space
4. Lateral subluxation
5. Vertical Physis
6. Speckled calcification
7. Wide femoral neck
Correct Answer - B:D:E
Ans. is, b' i.e., Lateral subluxation; 'd' i.e.,
Speckled calcification &'e' i.e., Wide femoral neck
Radiological findings in perthe's disease are :-

Increased medial joint space
Widening of femoral neck
Lateral extrusion (lateral subluxation)
Metaphyseal cysts and rarefaction of metaphysis
Horizontal physis with speckled calcifcation lateral to it
Fragmentation of femoral head with increased density (irregular
densities in the epiphysis).


4.
Late complication of supracondylar
fracture is ?

5. Ulnar nerve palsy
6. Cubitus varus
7. Cubitus valgus
8. Myositis ossificans
9. Volkmann's ischemic contracture
Correct Answer - B:D:E
Ans. 'b' 1.e., Cubitus Varus 'd' i.e., Myositis ossificans 'e'
i.e., Volkmann ischemic contracture Late Complications :

Occurring weeks to months after the fracture and include :
Malunion: - It is the commonest complication of supracondylar
fracture and results in cubitus varus (Gun stock deformity),
Cubitus valgus is rare and may occur occasionally in
posterolateral displacement.
Myositis ossificans and elbow stiffness.
Volkmann's ischemic contracture


.
Complication(s) of fracture of lateral
condyle humerus is/are ?

. Cubitus varus deformity
. Tardy ulnar nerve palsy
. Cubitus valgus deformity
. Median nerve injury
. Non-union
Correct Answer - A:B:C:E
Ans. is 'a' i.e., Cubitus varus deformity 'b' i.e., Tardy ulnar
nerve palsy; 'c' i.e., Cubitus valgus deformity; 'e' i.e., Non-
union Complications of Lateral condyle of humerus fracture

Lateral spur
Cubitus valgus
Rarely, cubitus varus
Tardy ulnar nerve palsy
Rarely avascular necrosis and myositis ossificans
Posterolateral instability and recurrent instability


.
Following are true regarding hangmans
fracture ?

6. Fracture of spinous process of C7
7. Fracture of C2 vertebra
8. Listhesis of the fracture vertebra
9. Knot is placed under nape of neck
10.
None
Correct Answer - B:C
Ans. is 'b' i.e., Fracture of C2 vertebra; 'c' i.e., Listhesis of the
fracture vertebra
Hangman's fracture
Hangman's fracture is bilateral fracture of the pars interarticularis of
axis (C2) with traumatic spondylolisthesis of axis (C2) over
C3 vertebrae. Thus Hangman's fracture is not simply a fracture,
but fracture dislocation of axis (C2).
The mechanism of injury is extension with distraction (in true, judicial
hangman's fracture) and hyper-extension, axial compression &.
flexion (in civilian injuries, which are now more common).
It is second most common type of Axis (C2) fracture, second only
to odontoid fractures.
Fatalaties are common, However, neurological deficit is unusual as
the fracture of posterior arch decompress the spinal cord.
Most of the fatalities occur at the scene of injury, acute post
admission mortality is low.
Successful healing of C2 traumatic spondylolisthesis is reported to
approach 95%. This is most commonly achieved with non-
operative


measures, even in the presence of displacement of pars
inter-articularis.
Undisplaced fractures are treated in a semi-rigid orthosis, and
displaced fracture are closed reduced & treated with halo-vest.
Occasionally, the hangman's fracture is associated with a C2/3 facet
dislocation. This is a severely unstable injury; open reduction and
stabilization is required.


180. True about keinbock's disease is/are ?
7. AVN of scaphoid
8. More common in males
9. May be associated with cerebral palsy
10.
Stage- I shows normal X-ray
11.
May cause carpal tunnel syndrome
Correct Answer - B:C:D:E
Ans. b) More common in males c) May be associated with
cerebral palsy d) Stage- I shows normal X-ray e) May cause
carpal tunnel syndrome
Keinbock's disease :
It is avascular necrosis of lunate bone. It is related to overuse and
ulnar negative wrist variance and may be associated with sickle
cell anemia, steroid abuse, gout and cerebral palsy.
It usually occurs in 2nd to 5 decade with male preponderance'
Patients complains of dorsal wrist pain, swelling, warmth, tenderness
over the radio lunate joint reduced ROM and decreased grip
strength.
Complications:
Stiffness
Loss of motion
Weakness
Carpal tunnel syndrome
Persistent pain
Instability
Degeneration in adjacent joints


181. True about location(s) of tumor is/are ?
8. Osteosarcoma is diaphyseal
9. Ewing sarcoma is diaphyseal
10.
Chondrosarcoma is metaphyseal
11.
Fibrosarcoma is diaphyseal
12.
Osteoclastoma is epiphyseal
Correct Answer - B:C:D:E
Ans. b) Ewing sarcoma is diaphyseal c) Chondrosarcoma is
metaphyseal d) Fibrosarcoma is diaphyseal e) Osteoclastoma
is epiphyseal
Epiphyseal tumor:
Chondroblastoma
Giant cell tumor (osteoclastoma)
Clear cell chondrosarcoma
Metaphyseal lesion:
Osteogenic sarcoma .
Unicameral (simple) bone cyst .
Aneurysmal bone cyst .
Fibrous cortical defect .
Chondrosarcoma
Osteochondroma
Enchondroma .
Osteoblastoma
Diaphyseal lesion:
Ewing sarcoma
Lymphomas
Fibrous dysplasia

Adamantinoma
Histiocytosis
Osteoid osteoma
Chondromyxoid fibroma
Fibrosarcoma
Fibrous cortical defect
Non ossifying fibroma


9.
A 28 years old lady presented with wrist
pain. X-ray wrist is showing lytic
eccentric lesion in lower end of radius
with soap bubble appearance. What is
the next plan of management ?

10.
Bone curettage and bone grafting
11.
Extended curettage with phenol
12.
Biopsy of the lesion
13.
Extended curettage with phenol and bone grafting
14.
Parathyroid and serum calcium levels measurement
Correct Answer - C
Ans. is'c' 1.e., Biopsy of the lesion
28 years female with lytic eccentric lesion in lower end of radius and
soap bubble appearance suggest the diagnosis of GCT.
Next plan of management would be biopsy of lesion to confirm the
diagnosis.


f)
True about the sites involved in
osteomyelitis is/are ?

f) Mainly involves metaphysis
g) Distal tibia is involved commonly
h) Proximal humerus is involved commonly
i) Proximal femur is involved commonly
j) Reaches to site by hematogenous routes
Correct Answer - A:C:E
Ans. is ,a, i.e., Mainly involves metaphysis;'c' i.e.,
Proximal humerus is involved commonly 'e' i'e', Reaches
to site by hematogenous route

Hematogenous osteomyelitis is the commonest form of osteomyelitis
and almost common source of bone and joint infection is
hematogenous.
It is caused most commonly by staphylococcus aureus.
Other causative organisms are streptococcus, pneumococcus, and
gram negative bacilli.
Infection by pseudomonas becomes proportionally much more
common in IV drug abuser. But, the most common organism
is staphylococcus aureus.


13. Compound palmar ganglion is caused by

?
f) Trauma
g) Overuse
h) RA
i) TB
j) SLE
Correct Answer - C:D
Ans. is'c' i.e., RA 'd' i.e., TB
Rheumatoid arthritis and tuberculosis are the commonest causes.
Tubercular and rheumatoid compound palmer ganglion is
characterized by presence of rice bodies, millet bodies and melon
seeds.


185. True about compound palmar ganglion ?
14.
Due to degeneration of flexor retinaculum
15.
Hour-glass in shape
16.
Surgery is the mainstay of treatment
17.
Intrasynovial steroid is the mainstay of treatment
18.
May be seen in rheumatoid arthritis
Correct Answer - A:D
Ans. is 'a' i.e., Due to degeneration of flexor retinaculum; 'd'
i.e., Intrasynovial steroid is the mainstay of treatment

Compound palmar ganglion is a misnomer because it is neither a
ganglion nor compound.
Chronic inflammation distends the common sheath of flexor tendons
both above and below the flexor retinaculum.
There is hourglass swelling, bulging above and below the flexor
retinaculum
Tubercular and rheumatoid compound palmer ganglion is
characterized by presence of rice bodise, millet bodies and
melon seeds
Rheumatoid arthritis and tuberculosis are the commonest causes.


f)
True about benign gestational
trophoblastic disease ?

k) Formed by placenta
l) Snow-Storm on USG
m)
Will not turn into malignant
n) Includes partial mole
o) Includes invasive mole
Correct Answer - A:B:D
Ans. is 'a' i.e., Formed by placenta; 'b' i.e., Snow-Storm on
USG; &'d' i.e., Includes partial mole

Gestational trophoblastic disease constitutes a diverse group of
lesions that includes abnormally formed placentas (hydatidiform
moles), benign nonneoplastic lesions, and gestational
trophoblastic neoplasms.
A characteristic pattern of multiple vesicles (snowstorm pattern) is
commonly seen with complete molar pregnancy.
Types:
Complete:
Morula differentiation to a chorion and amnion fails
Complete hydatidiform mole represents a proliferation of cells
containing 46 chromosomes of paternal origin only
Partial:
If a triploid karyotype is determined as 69, XXX, 69XXY, or 69 XYY
[6], then a partial mole can be confirmed.
Complete and partial mole have malignant potential


(J) Absolute contraindication of medical
abortion ?
21.
RHD
22.
Ectopic pregnancy
23.
Hypersensitivity to prostaglandins
24.
Corticosteroid therapy
25.
Porphyria
Correct Answer - B:C:E
Ans. is'b' i.e., Ectopic pregnancy;'c'i.e., Hypersensitivity
to prostaglandins; &'e'i.e., porphyria

Absolute contraindication of medical abortion:
Allergies to mifepristone/ misoprostol
Inherited porphyria
Chronic adrenal failure
Known or suggested ectopic pregnancy
Undiagnosed adnexal mass


o) What is the management of a woman
who had IUCD inserted and diagnosed to
have PID ?

(I) First give antibiotic then follow up
(J) First remove IUD then give antibiotics
(K)
First give antibiotic then remove IUD
(L)
Remove IUD and Simultaneously start antibiotic
(M)
No need for antibiotics just follow up
Correct Answer - A:B
Ans. is'a'i.e., First give antibiotic then follow up; > 'b'i.e.,
First remove IUD then give antibiotics

If an IUD user receives a diagnosis of PID, the IUD does not need to
be removed. However, the woman should receive treatment
according to these recommendations and should have close clinical
follow-up no clinical improvement occurs within 48-72 hours of
initiating treatment, providers should consider removing the IUD.
PID treatment regimens must provide empiric, broad spectrum
antibiotic coverage of likely pathogens


j)
A woman with amenorrhea is having
negative progesterone challenge test but
has bleeding on combined estrogen-
Progesterone challenge. What can be the
cause -

(E)
Anovulation
(F)
Asherman syndrome
(G)
Pregnancy
(H)
Pituitary tumor
(I) PCOD
Correct Answer - D
Ans. is 'd' i.e., Pituitary tumor
Negative progesterone challenge test - which rules out PCOD -
(otherwise too, PCOD is a cause of secondary amenorrhea).
When next step was done i.e., estrogen, progesterone combined
test - It comes out to be positive i.e., compartment I system
(uterus, endometrium and outflow tract) is normal if properly
stimulated by estrogen which rules out mullerian agenesis and
ashermann syndrome.
Positive estrogen progesterone combined test means the defect is in
the production of estrogen i.e., either ovaries, pituitary or
hypothalamus.


23. A 32 years women is presenting with
infertility with facial hair growth other
associated finding in the given patient
may be ?

k) Obesity
l) Diabetes mellitus
m)
High androgen levels
n) Hypertension
o) Acanthosis nigricans
Correct Answer - A:B:C:D:E
Ans. is 'a' i.e., Obesity; 'b'i.e., Diabetes mellitus; 'c' i.e.,
High androgen levels; 'd' i.e., Hypertension; &'e' i.e.,
Acanthosis nigricans

Infertility with hirsutism (facial hair growth) in a young
woman (32 years) suggests the diagnosis of PCOD (PCOS).

Young woman Central obesity
BMI > 30kg/cm2
Waist line > 88 cm
Oilgomenorrhoea, amenorrhoea Infertility (20%)
Hirsutism
Acanthosis nigricans due to insulin resistance.
Thick pigmented skin over the nape of neck, inner thigh and axilla
Most androgens from ovary
Increase fasting insulin > l0mlU/L


k)
Genes involved in high grade serous
carcinoma of ovary is ?

25.
K-RAS
26.
PTEN
27.
p53
28.
WT 1
29.
Beta - catenin
Correct Answer - C:D
Ans. is .c, i.e., p53; &'d' i.e., WTI
Two gene signatures of ovarian high grade serous
carcinoma are:-

WTl (Wilms tumour protein, a suppressor gene)
P53 (tumor suppressor gene) overexpression and mutation


k)
High risk marker(s) for serous ovarian
cancer are ?

g) p53
h) PAX2/8
i) Calretinin
j) All of the above
k) None of the above
Correct Answer - A:B
Ans. is 'a' i.e., p53; &'b' i.e., PAX2/8
The biomarkers of high grade serous ovarian cancer are
:- P53 over expression
PAX 2/8 (a mullerian marker) -+> a transcription factor that is
expressed in normal female genital tract epithelium (fallopian
tube, endometrium, endocervix)


k)
Which of the following can be used alone
as a method of contraceptive if OCP is
not started after menses?

24.
Copper T
25.
LNG - IUD ate
26.
Mifepristone
27.
Ulipristal acetate
28.
Norgestrel
Correct Answer - A:C:D
Ans. is.a, i.e., Copper T;'c' i.e., Mifepristone; &'d' i.e.,
Ulipristal acetate

Emergency contraceptives:
Levonorgestrel
Ulipristal acetate
Copper IUDs (Gold standard)
Mifepristone
Ethinyl estradiol
Ethinyl estradiol 50 ?g + norgestrel 0.25 mg


k)
All of the following are true regarding
breech presentation expect ?

. External ballottement can be elicited
. Uterine anomaly is the most common cause of
breech presentation
. Trial of labour is recommended if first fetus among twins
is breech because of smaller fetal weights of twins
. There is a better prognosis of fetus if preterm breech fetus
is delivered by caesarean section
. Lovset's maneuver can be done because of curved birth canal
Correct Answer - B:C
Ans. is .b, i.e., uterine anomaly is the most common cause of
breech presentation; & 'c' i'e', Trial of labour is recommended if
first fetus among twins is breech because of smaller fetal
weights of twins
ETIOLOGY:
Prematurity
Factors preventing spontaneous version:
Breech with extended legs
Twins
Oligohydramnios
Septate or bicornuate uterus
Short cord, relative or absolute
IUD of fetus.
Favourable adaptation:
Hydrocephalus

Placenta previa
Contracted pelvis
Cornu-fundal attachment of the placenta
Undue mobility of the fetus
Hydramnios,
Multipara with lax abdominal wall.
Fetal abnormality: Trisomies 13, 18, 21, anencephaly and myotonic
dystrophy
BIRTH INJURIES ASSOCIATED WITH BREECH
DELIVERY COMPLICATIONS

Brain damage
Spinal cord injury
Fetal distress
Umbilical cord prolapse
Seizures
Cerebral palsy
Compressed umbilical cord
Nerve damage
Umbilical cord wrapped around baby's neck
Oxygen deprivation


.
Condyloma acuminata is reported on
pap-smear as ?

k) Inflammatorycondition
l) Carcinoma
m)
Carcinoma in situ
n) LSIL
o) Normal
Correct Answer - D
Ans. is'd'i.e.LSIL
Cervical Precursor lesion associated with both low and high risk
HPV subtypes.
This category includes:
Flat mature LSIL (flat condyloma or CIN-I)
Mature Exophytic LSIL (exophytic condyloma, condyloma
acuminatum)
Extensive Exophytic LSIL (giant condyloma)
Immature Exophytic LSIL (immature condyloma, squamous
papilloma, papillary immature metaplasia)
Immature Flat Metaplastic LSIL


.
A high grade squamous intraepithelial
lesion is noted with pap, next
management includes ?

. Warthim's hysterectomy
. Local excision
. Colposcolic study and biopsy
. HPV DNA testing
. Liquid based cytology
Correct Answer - C
Ans' is'c'i.e., Colposcolic study and biopsy
For high grade intraepithelial lesions (HSIL), first step is to do
colposcopy and biopsy.
Moderate to severe dysplasia(CIN-II & CIN-III) (HSIL):
Treatment options are:
Local destructive methods
Cryosurgery
Fulguration/electrocoagulation .
Laser ablation
Excision of abnormal tissue
Cold knife conisation
Laser conisation
LLETZ
LEEP
NETZ
Surgery:
Therapeutic conisation

Hysterectomy
Hysterectomy with removal of vaginal cuff if carcinoma in situ
extends into vaginal vault


30. Screening test used in first trimester for
aneuploidy ?
k) PAPP-A &estradiol
l) PAPP-A & AFP
m)
PAPP-A &beta HCG
n) Beta HCG & inhibin
o) Estradiol& AFP
Correct Answer - C
Ans. is'c'i.e., PAPP-A &beta HCG
1st trimester aneuploidy screening:
Human chorionic gonadotropin (either intact or free (-hCG).
Pregnancy-associated plasma protein A (PAPP-A).
Fetal Down syndrome in 1st trimester:
Higher serum free beta-hCG level.
Lower PAPP-A levels.
Trisomy 18 & 13:
Lowered levels of both HCG PAPPP-A.
2nd trimester analytes:
Serum integrated screening.
Accuracy of aneuploidy detection:
Greater on combination with,
Sonographic NT measurement.


27. Tests for prenatal diagnosis of Down
syndrome ?
k) Nuchal fold thickness
l) Triple marker
m)
Karyotyping
n) Double marker
o) CVS (chronic venous sampling)
Correct Answer - A:B:C:D:E
Ans. is'a'i.e., Nuchal fold thickness;'b'i.e., Triple
marker;'c'i.e., Karyotyping;'d'i.e., Double marker; & 'e' i.e.,
CVS (chronic venous sampling)

Antenatal Screening for Downs syndrome :
Following methods are used :-
I ) Triple test : It includes
. Unconjugated estrogen (estriol) : decreased;
. Maternal serum alpha-feto protein (MSAFS) : decreased; and
. hCG z increased
New markers : These are
. Increased inhibin A in maternal blood; and
. Decreased PAPA
USG: It shows :
. Increased nuchal translucency in first trimester;
. Ductus venous Flow reversed; and
. Nasal bone hypoplasia.
Karyotyping: It can be done by chorionic villus sampling at 10-
12 weeks or amniocentesis at 16-18 weeks.




.
Most common cause of secondary
postpartum haemorrhage ?

28.
Trauma
29.
Infection
30.
Coagulation disorders
31.
Endometritis
32.
Retained Placenta
Correct Answer - B:D:E
Ans. is'b' i.e., Infection;'d'i.e., Endometritis; &'e'i.e., Retained
placenta
Secondary PPH:
Most common cause is infection, particularly in
association with:

. Retained placenta
. Obstructed labour causing necrosis of cervix
. Caeserion section & breakdown of uterine wound


200. True about carcinoma of vulva is/are ?
k) Adenocarcinoma is the most common type
l) May arise from Bartholin glands
m)
Smoking is a risk factor
n) Spread to inguinal lymph nodes
o) Present with pruritis
Correct Answer - B:C:D:E
Ans, is'b' i.e., May arise from Bartholin glands;'c' i.e.,
Smoking is a risk factor;'d'i.e., Spread to inguinal lymph
nodes & 'e' i.e., Present with pruritis.

Vulvar cancer is usually a squamous cell skin cancer, most often
occurring in elderly women.
Risk factors :
Vulvar intraepithelial neoplasia
HPV
Heavy cigarette smoking
Lichen sclerosis
Squamous hyperplasia
Squamous carcinoma of the vagina or cervix
Chronic granulomatous diseases
Spread:
By direct extension
Hematogenously
To the inguinal lymph nodes
From the inguinal lymph nodes to the pelvic and para-aortic lymph
nodes
Symptoms:

Palpable vulvar lesion:necrotic or ulcerated
Pruritus
Bleeding or a watery vaginal discharge
Melanomas


201. Feature of acute cervicitis is /are ?
33.
Fever
34.
Mucopurulent discharge
35.
Fullness in abdomen
36.
Dysuria
37.
Ulcer
Correct Answer - B:C:D
Ans. 'b' i.e., Mucopurulent discharge 'c' i.e., Fullness in
abdomen; & 'd' i.e., Dysuria
Acute Cervicitis :
Vaginal discharge
Congested & swollen cervix
Tenderness on touching cervix
Fullness in lower abdomen
Dypareunia and dysuria (due to concurrent urethral infection)
Vulvar or vaginal irritation
Cervicalfriability or bleeding.


202. True about adenomyosis is/are ?
g) Presence of myometrial tissue in endometrium
h) Presence of endometrium in peritoneal cavity
i) Mostly asymptomatic
j) May cause prolonged menstrual bleeding
k) Occurs in 4th to 5d decade
Correct Answer - C:D:E
Ans. is'c'i.e., Mostly asymptomatic;'d'i.e., May cause
prolonged menstural bleeding; &'e'i.e., Occurs in 46 to 56
decade Adenomyosis

A condition characterized by the presence of ectopic glandular
tissue found in muscle.
It usually refers to ectopic endometrial tissue (the inner lining of the
uterus) within the myometrium (the thick, muscular layer of the
uterus).
The condition is typically found in women between the ages of 35
and 50.
Patients with adenomyosis can have dysmenorrhea & menorrhagia.
In adenomyosis, basal endometrium penetrates into hyperplastic
myometrial fibers.
Therefore, unlike functional layer, basal layer does not undergo
typical cyclic changes with menstrual cycle.


203. Not done in Category 2 NST ?
g) Changes in maternal position
h) Oxygen supplementation
i) Avoiding maternal pushing
j) Uterine stimulation
k) Intravenous fluid administration
Correct Answer - D
Ans. is'd'i.e., Uterine stimulation
Intrauterine resuscitative measures for Category II NST:
Supplemental oxygen
Maternal position changes
Intravenous fluid administration
Correction of hypotension
Reduction or discontinuation of uterine stimulation
Administration of uterine relaxant
Amnioinfusion
Changes in second stage breathing and pushing techniques
NST Category II :
These are indeterminate tracings.
Not predictive of fetal acid - base status.
Cannot be classified as category I or III.
Require continued surveillance and re-evaluation. meet the
"reactivity"
Except for patterns that are sinusoidal, category III tracings require
the combination of absent variability plus one or more additional
finding. Such tracings are classified as category II.


31. Which of the following are not true
regarding genital ulcerations ?
g) Syphilis has a large single nontender ulcer with
indurated margins
h) Multiple erythematous lesions with tender lymphadenaopathy
is seen in LGV
i) Beefy red ulcer with indurated ulcer is seen in donovanosis
j) Multiple painful bleeding ulcer with tender lymphadenopathy
in Chancroid
k) Multiple painful ulcers in HSV
Correct Answer - A:C:D:E
Ans. (A) Syphilis has a large single nontender ulcer with
indurated margins (C) Beefy red ulcer with indurated ulcer
is seen in donovanosis (D) Multiple painful bleeding ulcer
with tender lymphadenopathy in Chancroid (E) Multiple
painful ulcers in HSV

[Ref: Khopkar's 6/e p. 232]
Lesion of primary syphilis:
Chancre (Hard chancre) - punched out ulcer
Lesion characteristics:
32. Single lesion
33. Painless
34. Avascular(non-bleeding)
35. Firm induction
36. Lymphadenopathy which is painless, firm and nonsuppurative.
Sites of involvement are penis in heterosexual males; rectum, anal


canal, mouth in homosexual males; and cemix and labia in females.
Lesions of LGV:
First stage (Primary LGV) : - Self limited, Single, asymptomatic,
painless, nonbleeding genital ulcer.
Secondary stage:
Painful inguinal lymphadenopathy (Remember ) Wcer is painless but
lymphadenopathy is tender & painful).
Swollen lymph nodes coalesce to form bubos,i.e., matted lymph
nodes.
Buboes may rupture to form discharging sinus.
Groove's sign > Enlarge lymph nodes both above and below the
inguinal ligament.
Tertiary LGV (genitorectal syndrome) : - Characterized By
proctocolitis.
Lesions of Donovanosis:
. Painless
g) Bleeding with red granulation tissue
h)
Indurated
i) Red & velvety (beefy red)
Subcutaneous granulomas of inguinal region in Donovanosis looks
like enlarged lymph node, but these are not enlarged lymph nodes.
Therefore, these are known as pseudo buboes.
Sites of lesions are genitalia (90%), inguinal (10%o) and anal
regions.
Complications of Donovanosis are pseudo elephantiasis, phimosk,
paraphimosis.
Characterized by painful ulcers, bubo formation and painful inguinal
lymphadenopathy.
. Multiple ulcers.
37. Non-indicated or soft induration
38. Paintul (Tender)
39. Bleed easily
40. Undermined, sloughed erythematous edges
41. Painful suppurative inguinal lymphadenopathy
Lesion of HSV:
Often asymptomatic when symptomatic it present as multiple


vesicles, very painful,
bilateral painful inguinal lymphadenopathy.
Characteristic feature of herpes genitalis is its frequent recurrence.


g) A young male developed ulceration over
shaft which bleeds easily on touch, is
tender with bilateral lymphadenopathy.
The following are true regarding the
condition -

k) Bilateral drainage of lymph nodes is essential
l) School of fish appearance is seen with smear microscopy
m)
Medical treatment with antibiotics is mainstay of treatment
n) Azithromycin is the drug of choice
o) It is a case of Hard-chancre
Correct Answer - B:C:D
Ans. (B) School of fish appearance is seen with smear
microscopy (C) Medical treatment with antibiotics is
mainstay of treatment (D) Azithromycin is the drug of choice

Chancroid is STD characterized by painful ulcers, bubo formation
and painful inguinal lymphadenopathy.
Caused by H.ducreyi, a gram negative coccobacilli which is
arranged in parallel chains giving a "School of fish" or "railroad
track" appearance.
Incubation period of chancroid is 1-7 days.
Clinically it is characterized by: -
. Multiple ulcers.
g) Non-indicated or soft induration
h)
Painful (Tender)
i) Bleed easily

. Undermined, sloughed erythematous edges
36. Painful suppurative inguinal lymphadenopathy
Diagnosis & Treatment
Gram's staining of swab from the lesion may reveal a predominance
of characteristic gram-negative coccobacilli.
An accurate diagnosis relies on cultures of H.ducreyi from the lesion.
Azithromycin is the DOC for treatment.
Ceftriaxone, Ciprofloxacin, erythromycin are alternatives.


g) Which of the following organism is
involved in causation of Pityriasis
rosaceae?

41.
Propionibacterium
42.
Pityrosporum ovale
43.
Malasezia furfur
44.
Human herpes virus-6
45.
Human herpes virus-7
Correct Answer - D:E
Ans. (D) Human herpes virus-6 (E) Human herpes virus-7
P. rosea is a common scaly disorder, occurring usually in children
and young adults (10-j5 years). Characterized by round/oval pink
brown patches with a superficial, centrifugal scale, distributed
over trunk in a Christmas tree pattern.
The disease is thought to be viral disease, is self limiting, and
subsides in 6-12 weeks.
The exact etiology is not known, but it is considered to be a viral
disease; Human Herpesvirus 6 (HHV 6) and}Jtrrnian Herpesvirus
7 (HHV 7) may play a role.


207.


Spongiosis involves which part of skin ?
g) Dermis
h) Epidermis
i) Stratum spinosum
j) Stratum corneum
k) Prickle cell layer
Correct Answer - B:C:E
Ans. (B) Epidermis (C) Stratum spinosum (E) Prickle cell layer
Spongiosis involves stratum spinosum (prickle cell layer) of the
Epidermis.


208. Koebner's phenomenon is seen in ?
38.
Pemphigus vulgaris
39.
DLE
40.
Lichen planus
41.
Acne rosae
42.
Genital warts
Correct Answer - B:C:E
Ans. (B) DLE (C) Lichen planus (E) Genital warts


k)
Anaesthetic used for induction in

pediatric surgery is ?
k) Propofol
l) Thiopentone
m)
Ketamine
n) Diazepam
o) Etomidate
Correct Answer - A:B:E
Ans. (A) Propofol (B) Thiopentone (E)
Etomidate Anaesthetics in Pediatric patients:
Induction

Inhalational induction:
Inhalational agent with mask - Induction method of choice in
children.
Sevoflurane - Induction agent of choice in children.
Used in N2O +O2 gas mixture.
Halothane - 2nd Induction agent of choice.
Intravenous induction:
THiopental/propofol (Outpatient surgery).
Ketamine - preferred in children with hypovolemia.
Etomidate - preferred in children with unstable cardiovascular status


210. Atracurium is metabolized by -
g) Conjugation
h) Hoffman degradation
i) Pseudocholineaterase
j) Methylation
k) None
Correct Answer - B
Ans. B. Hoffman degradation
The unique feature of atracurium is inactivation in plasma by
spontaneous non enzymatic degradation (Hofmann
elimination).
Consequently its duration of action is not altered in patients with
hepatic/renal insufficiency or hyperdynamic circulation ---> Hence,
preferred muscle relaxant for such patients as well as for
neonates and the elderly.
Atracurtum is metabolised to laudanosine that is responsible for
seizures.
Cause histamine release > Hypotension, bronchoconstriction &
flushing.


45. Ventilator associated complication(s)
is/are ?
g) Barotrauma
h) Subglottic stenosis
i) Pneumoperitoneum
j) Paralytic ileus
k) Increased cardiac output
Correct Answer - A:B:C:D
Ans. (A) Barotrauma (B) Subglottic stenosis
46. Pneumoperitoneum (D) Paralytic ileus [Ref
Essentials of anesthetic emergencies p. 123]

Complications of mechanical ventilator:
Barotrauma - Cause pneumothorax, pneumomediastinum,
bronchopleural fistula, pneumopericardium/cardiac
tamponade, Pneumoperitoneum, systemic air embolism and
pulmonary embolism.
Hemodynamic complications
Nosocomial infections: Pneumonia, UTI
Acid-base disturbances - Respiratory alkalosis due to CO2 washout.
Water retention.
GIT - Mainly paralytic ileus.


g) Headache following dural puncture,
treatment is:
g) ACTH
h) Clonidine
i) Steroids
j) Blood
k) Caffeine
Correct Answer - A:C:E
Ans. (A) ACTH (C) Steroids (E) Caffeine
Ref: Morgan's 4,h/e p. 297, Lee's 13h/e p.
509, 510; www.cochrane.org
Post dural puncture headache:
Due to CSF leak from a dural defect & decreased ICT.
Most common complication of spinal anesthesia.
Typical location is bifrontal or occipital.
Onset
Usually 12-72hotrs following the procedure.
Lasts for 7-10 days.
Management:
Use of small bore needle can prevent pDpH.
Conservative treatment:
Analgesics (NSAIDs), oral or i.v. fluids.
Drugs: Cosyntropin, caffeine, hydrocortisone, gabapentin,
theophylline, sumatriptan, pregabalin and ACTH.


213. Pre-anaesthetic medication is given to ?
48.
Reduce anxiety and fear
49.
Reduction of secretion of saliva
50.
To produce amnesia
51.
To prevent undesirable reflexes
52.
Prevent vomiting
Correct Answer - A:B:C:D
Ans. (A) Reduce anxiety and fear (B) Reduction of secretion
of saliva (C) To produce amnesia (D) To prevent undesirable
reflexes

[Rel KDT 6h/e p. 378]
Preanaesthetic medication:
Aims:
k) Relief of anxiety and apprehension preoperatively and to
facilitate smooth induction.
. Amnesia for preoperative and postoperative events.
45. Supplement analgesic action of anaesthetics and potentiate them.
46. Decrease secretions and vagal stimulation (undesirable reflex).
47. Antiemetic effect extending into postoperative period.
48. Decrease acidity and volume of gastric juice so that it is
less damaging if aspirated.


214. Methods of regional anaesthesia is/are ?
k) Bier's block
l) Spinal anaesthesia
m)
Rapid sequence induction
n) Conscious sedation
o) Surface anaesthesia
Correct Answer - A:B:E
Ans. (A) Bier's block (B) Spinal anaesthesia (E)
Surface anaesthesia

[Ref: Morgan 4/e p. 269-270]
Regional anaesthesia (Local anaesthesia):
Methods are:
50. Topical anaesthesia (surface anaesthesia)
51. Infiltration anaesthesia
52. Intravenous regional anaesthesia (Bier's block)
53. Conduction block (either field block or nerve block)
54. Spinal anaesthesia
55. Epiduralanaesthesia


g) If we increase the depth of chest
compression in CPR, it causes -
k) Decreased mortality
l) Increased brain perfusion
m)
Increased aortic pressure
n) Rib fracture
o) Hemothorax
Correct Answer - A:B:C:D:E
Ans. (A) Decreased mortality (B) Increased brain perfusion
g) Increased aortic pressure (D) Rib fracture (E) Hemothorax
Increasing the depth of chest compression also carry an
increased risk of complications like :-
Rib and / or sternal fracture
InjurY to diaPhragm or lung
Pneumothorax, pneumomediastinum, pneumopericardium
Hemothorax


k)
Hypotensive shock refractory to fluid,
what is contraindicated ?

k) Ketamine
l) Atropine
m)
Fentanyl
n) Thopentone
o) Etomidate
Correct Answer - C:D
Ans. (C) Fentanyl (D) Thopentone
[Ref: Wroerlee textbook of anaesthesia p.54]
In hypotensive patients, no sedative, hypnotic or opiate should be
given.
Fentanyl is an opiate and thioPentone is a sedative (barbiturate).
Ketamine increases cardiac output and blood pressure - Intravenous
anaesthetic of choice in shock.
Etomidate produces little cardi-ovascular anil respiratory depression
.
Agent of choice for cardiovascular surgeries (bypass
aneurysms, valve surgery).
Etomidate is most cardiostable inducing agent.
If hypotension is due to bradycardia --> Atropine is the drug of
choice.


217. Egg shell calcification is seen in ?
g) Silicosis
h) Sarcoidosis
i) Lymphoma after treatment
j) Aspergilloma
k) TB
Correct Answer - A:B:C:E
Ans. (A) Silicosis (B) Sarcoidosis (C) Lymphoma after treatment
(E) TB
Calcification patterns on chest radiograph
Egg-shell calcification of lymph nodes:
Defined as she like calcifications up to 2 mm thick in periphery of at
least two lymph nodes in at least one of which, the ring of
calcification must be complete and one of the affected lymph
nodes must be at least I cm in maximum diameter.
Important causes are : -
Progressive massive fibrosis (PMF)
Coccidioidomycosis
Tuberculosis
Silicosis (m.c. cause)
Coal worker's Pneumoconiosis
Sarcoidosis
Blastomycosis
Histoplasmosis
Scleroderma
Lymphoma following radiation
Amyloidosis




56. Hilar lymph node calcification is seen in

?
g) Silicosis
h) Amyloidosis
i) Beryliosis
j) Asbestosis
k) Scleroderma
Correct Answer - A:B:E
Ans. (A) Silicosis (B) Amyloidosis (E) Scleroderma
Irregular central popcorn calcification:

It is a cluster of sharply defined irregularly lobulated calcification
usually in pulmonary nodule.
It is characteristic of hamartoma.
Laminated or central pattern:
Granuloma
Punctate Pattern:
Tuberculoma or coccidioidomycosis


(J) Radiological signs of intestinal
perforation is/are-
57.
Pneumoperitoneum
58.
Regler's sign
59.
Caterpillar sign
60.
Bear sign
61.
Football sign
Correct Answer - A:B:E
Ans. (A) Pneumoperitoneum (B) Regler's sign (E) Football sign
Perforation results in pneumoperitoneum:
Best view to see PneumoPeritoneum is chest x-ray in erect position
which detects air under the dome of the diaphragm.
1-2 ml of free air can be detected under the right dome (between the
liver anil right ilome of diaphragm) provided the patient is made to
stand or sit at least 10 minutes prior to taking radiograPh.
Visualization of falciform ligament due to the presence of air on either
side of the ligament - Falciform ligament sign.
Football sign:
Presence of large quantities of air which form an interface with free
intraperitoneal fluid.
Regler's sign or double wall sign:
Visualization of both aspects of bowel wall due to the presence of
intraluminal as well as extraluminal air.
Cupola sign:
Large amount of gas under the diaphragm. It should be noted that
air is visualized below the central tendon of diaphragm not below
the


dome as occur in upright x-rays.
Inverted 'V' sign:
Lateral umbilical ligament is visualized in lower abdomen.
Triangle sign (Doge's cap sign):
Triangular (doge's cap), crescent shaped or semicircular collection
of air in the Morison'spouch.


220. Unit(s) of absorbed radiation ?
g) Rad
h) Gray
i) Curie
j) Rem
k) Sievert
Correct Answer - A:B
Ans. (A) Rad (B) Gray
[Ref: Bhadhury 2d/e p. 197 & Internet source]
SI unit
Conventional unit
Radioactivity
Becquerel (Bq) Curie (Ci)
1 Bq = 1 disintegration per second
1 Ci = 3.7 ? 1010 disintegrations per
second =37 GBq
Absorbed dose
Gray (Gy)
rad
1 Gy = 1 J/kg = 100 rad
Effective dose
Sievert (Sv)
rem
1 Sv = 100 rem
Linear energy transfer Newton (N)
keV/m
1 N = 1 J/m
1 keV/m = 1.6 ? 10-13 N


.
Isotope(s) used for brachytherapy is/are

?
g) Radon -222
h) Radium -226
i) Iodine-125
j) Cobalt - 60
k) Cesium -137
Correct Answer - A:B:C:D
Ans. (A) Radon -222 (B) Radium -226 (C) Iodine-125 (D) Cobalt -
60

[Ref: Prez & Brandy's Radiation Oncologist 5th/e p, 54]
Three main types of radiotherapy depending upon the position of the
source of radiation.
External beam radiotherapy (EBRT) or Teletherapy:
X-rays beams (Linear acceleration).
Gamma rays: - Cobalt - 60 beam or Cesium 137.
Particulate beams
Internal radiotherapy or brachytherapy:
In brachytherapy, the radiation source in close contact with tumor.
The principle is to use an Intra or fuxta lesional radiation implant to
irradiate the tumor in vicinity. Sealed source in placed within or near
the tumor i.e., Short distance therapy.
Subtypes:
Interstitial
Removable/Temporary sources
Permanent sources
or implants
or implants
Iridium - 192 (Ir - 192)
Cesium - 131 (Cs - 131)


Cesium - 137 (Cs - 137)
Yttrium
Cobalt - 60 (Co - 60)
Gold - 198 (Au - 198)
Californum
125I
Radium - 226 (Ra - 226)
Radon - 222 (Rn - 222)
Tantalum
Pallidum - 103 (Pd - 103)
Note : Co - 60 and Cs - 137 are used for both teletherapy
and brachytherapy.

Intracavitary:
Most important use of intracavitary radiation is in patients with
cancer of the cervix after external radiation.
It is also used in cancer of uterus, esophageal cancer and lung
cancer.
Removable
Permanent
Radium
Colloidal radioactive gold
Cesium ? 137 Yttrium
Coblat ? 60
Radioactive iodine (131I)
Mould:
Used in penis carcinoma
High dose radiation:
Most common radioisotope used for HDR brachytherapy is iridium-
I92.
Other isotopes which have been used are Cobalt-60 and cesium-
137


g) Isotope(s) used for metastatic bone pain
is/are ?
(K)
Strontium -89
(L)
Samarium-153
(M)
Phosphorus - 32
(N)
Thallium
(O)
Selenium
Correct Answer - A:B:C
Ans. (A) Strontium -89 (B) Samarium-153 (C) Phosphorus -
32
[Ref: Radiation oncology th/e P. 141]
Radiopharmaceuticals (radioactive isotopes) used for metastatic
bone pain are strontium (Sr - 89), Samarium (Sm - 153),
rhenium (Re - 186), Phosphorus-32 and Tin- 117 (Sn- 1 77 ).


56. Drug(s) used for radiation protection
is/are ?
k) Metronidazole
l) Amifostine
m)
Actinomycin -D
n) Pentoxiphylline
o) Hydroxyurea
Correct Answer - A:D
Ans. (A) Metronidazole (D) Pentoxiphylline
[Ref: Orford oncologist 2nd/e p. 462]
Radiosensitizers:
Radiosensitizers are compounds that apparently promote fixation of
the free radicals produced by radiation damage at molecular level.
A) Hypoxic cell radiosensitizers:
Nitroimidazoles: Metronidazole, Misonidazole, Etanidazole,
Nimorazole, Pimonidazole
Anticancers:- Actinomycin D (Dactinomycin), Bleomycin, Cisplatin,
Doxorubicin, 5-FU,
Fludarabine, Gemcitabine, Hydroxyurea, Paclitaxel, mitomycin- C,
Topotecan, Vinorelbine.
Hyperbaric oxygen (most potent)
m) Non hypoxic cell radiosensitizers:
Haloginated pyrimidines:- BUdR, IUdR.

Cisplatin and 5-FU are two of the most commonly used
radiosensitizing agents.


58. Cannabis abuse may be associated with

?
k) Psychosis
l) Schizophrenia
m)
Anxiety
n) Flash-back
o) OCD
Correct Answer - A:B:C:D
Ans. (A) Psychosis (B) Schizophrenia (C) Anxiety (D)
Flash-back

[Ref Kaplan & Sadock's p.420]
Cannabis intoxication:
Most common physical effects are red eye (conjunctival injection)
and mild tachycardia.
Increased appetite ("the munchie")
Dry mouth
Lightheadedness
Euphoria
Sense of floating in air
Derealization
Depersonalization
Synesthesia (stimulation of one sensory modality produces
sensation of other modality).


61. Treatment madality(ies) used in mania
is/are ?
k) ECT
l) Lamotrigine
m)
Mood stabilizer
n) Olanzapine
o) Valproate
Correct Answer - A:C:D:E
Ans. (A) ECT (C) Mood stabilizer (D) Olanzapine (E) Valproate
Treatment of bipolar disorder:
Treatment of acute mania:
. Lithium
. Valproate
. Carbamazepine, Oxcarbazepine
. Atypical 6 typical antipsychotics: Olanzapine, risperidone, quetiapine,
ziprasidone, aripiprazole, Haloperidol, Chlorpromazine.
g) Benzodiazepines
Treatment of acute bipolar depression:
. Antidepressants with a mood stabilizer - Lamotrigine or ziprasidone.
Maintenance treatment:
k) Lithium (drug of choice)
l) Carbamazepine.
m) Valproate, other drugs which can be used are topiramate,
Iamotrigine, atypical antipsychotics (aripiprazole, olanzapine,
quetiapine, risperidone, Clozapine) and Gabapentin.
Treatment of rapid cycling:

. Valproate is the drug of choice.
. Other drugs used are Carbamazrpine, Lithium, lamotrigine.


k)
True about somatoform disorders and
somatization disorder?

k) There are physical symptoms without physical basis
l) Somatization disorder is characterized by multiple
physical symptoms
m)
Conversion disorder is a somatization disorder
n) Hypochondriasis is a somatoform disorder
o) There are pain and GI symptoms in somatization disorders
Correct Answer - A:B:D:E
Ans. (A) There are physical symptoms without physical basis
68. Somatization disorder is characterized by multiple physical
symptoms (D) Hypochondriasis is a somatoform disorder


k) There are pain and GI symptoms in somatization disorders

ACCORDING TO DSM -TV CLASSIFICATION:
Somatoform disorders are characterized by repeated presentation
with physical symptoms but without any physical basis.
Somatoform disorders are: - (i) Somatization disorder, (ii)
Conversion disorder, (iii) Hypochondriac, (iv) Body dysmorphic
disorder ft) Somatization pain disorder.
Somatization disorder:
Multiple recurrent somatic symptoms of long duration caused by
psychological basis and no physical illness.
Diagnostic criteria:
Four pain symptoms - Pain in Head (headache), abdomen, back,
joint, extremities,
chest, rectum, during menstruation or sexual intercourse or


urination.
Two gastrointestinal symptoms - Nausea, bloating, vomiting,
diarrhea.
One sexual symptom - Erectile dysfunction (Impotence), ejaculatory
dysfunction.
One pseudoneurological symptom - Mainly glove & stocking
anesthesia, Paresthesia,
SOMATIC SYMPTOMS & RELATED DISORDERS (DSM-V):
DSM-5 has replaced somatoform disorders (of DSM-IV) with somatic
symPtoms and related disorders.
Important disorders in this group are :-
k) Somatic symptom disorder (complex somatic symptom disorder)
l) Illness anxiety disorder
m) Conversion disorder (functional neurological disorder )
n)
Factitious disorders
o)
Other specified somatic symptoms & related disorders
(e.g' Pseudocyesis)
Note: Body dysmorphic disorder is replaced along with OCD
& related disorder,


227. Legal psychiatric conditions is/are ?
(K)
Mania
(L)
Delirium
(M)
OCD
(N)
Delusion
(O)
Phobia
Correct Answer - A:B:C:D:E
Ans. (A) Mania (B) Delirium (C) OCD (D) Delusion (E) Phobia
The legal term 'mentally ill person' as used in Mental Health Act
1987 (of India), means a person who is in need of treatment by
reason of any mental disorder other than mental retardation.
Psychoses
66. Organic Psychoses

. Dementia

. Drug induced psychosis : Alcohol, Heroin, Cannabis, LSD, Cocaine.

. Confusional states and psychosis following epilepsy, pregnancy and
childbirth, and trauma.

. Delirium

67. Functional psychoses
. Schizophrenia
. Paranoid states : Delusions
k) Affective disorders (Mania; depression)
B) Neurotic disorders
. Anxiety neurosis
. Phobia
. Hysteria
. OCD

C) Various organic disorders


228. Feature(s) of catatonic schizophrenia ?
g) Waxy flexibility
h) Automatic obedience
i) Somatic passivicity
j) Rigidity
k) Mutism
Correct Answer - A:B:C:D:E
Ans. (A) Waxy flexibility (B) Automatic obedience (C)
Somatic passivicity (D) Rigidity (E) Mutism

[Ref: Neeraj Ahuja e p. 63; Essentials of clinical psychiatry 4th/e
p. 635]
Features of catatonic schizophrenia may be
:
-l) Excited catatonia:
Characterized by increased psychomotor activity, i.e. restlessness,
agitation, excitement, aggressiveness, violence. The impulsive
activity occurs in response to hallucinations and delusions.
2) Stuporous (retarted catatonia) :
Characterized by extreme retardation of psychomotor activity.It
includes mutism, rigidity, negativism, posturing, stupor, echolalia,
echopraxia, catalepsy (waxy flexibility), ambitendency,
gegenhalten, stereotypes, stupor, mannerism, Grimacing,
automatic obedience.


229. True about Alzheimer's disease?
72.
Early involvement of short term memory
73.
Late involvement of long term memory
74.
Agnosia
75.
Clouding of consciousness
76.
Depression
Correct Answer - A:B:C:E
Ans. (A) Early involvement of short term memory (B) Late
involvement of long term memory (C) Agnosia (E)
Depression
[Ref: Kaplan 6 Saddock's 10'h/e p. 331]
Alzheimer's disease usually presents in 5th or 6th decade.
Gradually progressive disease.
Features:
Memory impairment:
. Gradual development of forgetfulness is the major symptom.
k) Initially short term memory (memory for recent events and
learning new information) is involved.
. Long term memory is involved late.
69. Episodic type declarative memory is affected mostly.
70. No impairment in
consciousness. Associated
disturbance
-
Disorientation, aphasia, apraxia, agnosia (anosognosia,
prosopagnosia) acalculia, lexical anomia, judgement
imPairment.
Psychiatric symptoms:
Visual hallucination, Depression and anxiety.


invalid question id

This post was last modified on 11 August 2021